SlideShare une entreprise Scribd logo
1  sur  58
Télécharger pour lire hors ligne
January 27, 2005 11:45       L24-CH08           Sheet number 1 Page number 337                       black



                                                                     CHAPTER 8
                                  Principles of Integral Valuation

              EXERCISE SET 8.1
                                                  1                1          1
               1. u = 4 − 2x, du = −2dx, −                u3 du = − u4 + C = − (4 − 2x)4 + C
                                                  2                8          8

                                            3        √
               2. u = 4 + 2x, du = 2dx,                  u du = u3/2 + C = (4 + 2x)3/2 + C
                                            2

                                            1                            1            1
               3. u = x2 , du = 2xdx,              sec2 u du =             tan u + C = tan(x2 ) + C
                                            2                            2            2

               4. u = x2 , du = 2xdx,       2     tan u du = −2 ln | cos u | + C = −2 ln | cos(x2 )| + C

                                                                         1   du    1              1
               5. u = 2 + cos 3x, du = −3 sin 3xdx,                  −          = − ln |u| + C = − ln(2 + cos 3x) + C
                                                                         3    u    3              3

                         2        2    1           du    1             1     2
               6. u =      x, du = dx,                  = tan−1 u + C = tan−1 x + C
                         3        3    6         1 + u2  6             6     3

               7. u = ex , du = ex dx,           sinh u du = cosh u + C = cosh ex + C

                                    1
               8. u = ln x, du =      dx,         sec u tan u du = sec u + C = sec(ln x) + C
                                    x

               9. u = tan x, du = sec2 xdx,               eu du = eu + C = etan x + C

                                            1             du    1             1
              10. u = x2 , du = 2xdx,                √         = sin−1 u + C = sin−1 (x2 ) + C
                                            2            1−u 2  2             2

                                                                 1                    1 6         1
              11. u = cos 5x, du = −5 sin 5xdx,             −            u5 du = −      u + C = − cos6 5x + C
                                                                 5                   30          30
                                                                             √
                                                            du            1 + 1 + u2            1 + 1 + sin2 x
              12. u = sin x, du = cos x dx,                √       = − ln            + C = − ln                +C
                                                          u u2 + 1            u                     sin x

                                                      du
              13. u = ex , du = ex dx,           √          = ln u +            u2 + 4 + C = ln ex +         e2x + 4 + C
                                                     4 + u2

                                            1                                               −1
              14. u = tan−1 x, du =             dx,              eu du = eu + C = etan           x
                                                                                                     +C
                                         1 + x2

                         √                 1                                                     √
              15. u =        x − 1, du = √    dx,            2       eu du = 2eu + C = 2e         x−1
                                                                                                        +C
                                        2 x−1

                                                            1                       1                  1
              16. u = x2 + 2x, du = (2x + 2)dx,                      cot u du =       ln | sin u| + C = ln sin |x2 + 2x| + C
                                                            2                       2                  2

                         √           1                                                   √
              17. u =        x, du = √ dx,            2 cosh u du = 2 sinh u + C = 2 sinh x + C
                                    2 x

                                                                              337
January 27, 2005 11:45          L24-CH08                 Sheet number 2 Page number 338               black



             338                                                                                                               Chapter 8


                                           dx            du    1       1
             18. u = ln x, du =               ,            2
                                                             =− +C =−      +C
                                            x            u     u      ln x

                         √            1                           2 du                           2 −u ln 3        2 −√x
             19. u =          x, du = √ dx,                            =2     e−u ln 3 du = −        e     +C =−      3 +C
                                     2 x                           3u                           ln 3             ln 3

             20. u = sin θ, du = cos θdθ,                         sec u tan u du = sec u + C = sec(sin θ) + C

                             2          2                     1                     1             1    2
             21. u =           , du = − 2 dx,            −          csch2 u du =      coth u + C = coth + C
                             x         x                      2                     2             2    x

                              dx
             22.       √            = ln x +             x2 − 4 + C
                             x2 − 4

                                                                      du      1   2+u        1   2 + e−x
             23. u = e−x , du = −e−x dx,                      −            = − ln     + C = − ln         +C
                                                                     4−u 2    4   2−u        4   2 − e−x

                                           1
             24. u = ln x, du =              dx,             cos u du = sin u + C = sin(ln x) + C
                                           x

                                                              ex dx                du
             25. u = ex , du = ex dx,                    √            =       √          = sin−1 u + C = sin−1 ex + C
                                                              1 − e2x             1 − u2

                                                    1
             26. u = x−1/2 , du = −                     dx,          −     2 sinh u du = −2 cosh u + C = −2 cosh(x−1/2 ) + C
                                                  2x3/2

                                                     1         du     1                  1             1
             27. u = x2 , du = 2xdx,                                =        sin u du = − cos u + C = − cos(x2 ) + C
                                                     2        csc u   2                  2             2

                                                                   2du
             28. 2u = ex , 2du = ex dx,                       √           = sin−1 u + C = sin−1 (ex /2) + C
                                                                  4 − 4u2

             29. 4−x = e−x
                         2         2
                                       ln 4
                                              , u = −x2 ln 4, du = −2x ln 4 dx = −x ln 16 dx,
                         1                           1 u           1 −x2 ln 4         1 −x2
                   −              eu du = −              e +C =−       e      +C =−       4 +C
                       ln 16                       ln 16         ln 16              ln 16

                                                                    1 πx ln 2        1
             30. 2πx = eπx ln 2 ,                 2πx dx =               e    +C =        2πx + C
                                                                  π ln 2           π ln 2

                                                                               1 2       1
             31. (a) u = sin x, du = cos x dx,                           u du =  u + C = sin2 x + C
                                                                               2         2
                                                         1                     1              1
                   (b)          sin x cos x dx =                  sin 2x dx = − cos 2x + C = − (cos2 x − sin2 x) + C
                                                         2                     4              4
                         1                          1                              1 1
                   (c) − (cos2 x − sin2 x) + C = − (1 − sin2 x − sin2 x) + C = − + sin2 x + C,
                         4                          4                              4 2
                       and this is the same as the answer in part (a) except for the constants.

                                                 1              1
             32. (a) sech 2x =                        =                                  (now multiply top and bottom by sech2 x)
                                              cosh 2x   cosh2 x + sinh2 x
                                                sech2 x
                                       =
                                              1 + tanh2 x
January 27, 2005 11:45       L24-CH08         Sheet number 3 Page number 339             black



              Exercise Set 8.2                                                                                           339


                                                sech2 x
                    (b)       sech2x dx =                 dx = tan−1 (tanh x) + C,        or, replacing 2x with x,
                                              1 + tanh2 x

                              sechx dx = tan−1 (tanh(x/2)) + C

                                      1        2       2ex
                    (c) sech x =           = x      = 2x
                                    cosh x  e + e−x  e +1
                                                 ex
                    (d)       sech x dx = 2          dx = 2 tan−1 (ex ) + C
                                               e2x+1

                           sec2 x        1              1
              33. (a)             =              =
                           tan x    cos2 x tan x   cos x sin x
                                       1           1         1 sec2 x                        1
                    (b) csc 2x =           =               =          , so     csc 2x dx =     ln tan x + C
                                    sin 2x   2 sin x cos x   2 tan x                         2
                                      1          1
                    (c) sec x =           =              = csc(π/2 − x), so
                                    cos x   sin(π/2 − x)
                                                               1
                              sec x dx = csc(π/2 − x) dx = − ln tan(π/2 − x) + C
                                                               2



              EXERCISE SET 8.2
               1. u = x, dv = e−2x dx, du = dx, v = − 1 e−2x ;
                                                      2
                                     1               1 −2x       1       1
                         xe−2x dx = − xe−2x +          e   dx = − xe−2x − e−2x + C
                                     2               2           2       4

                                                        1 3x                  1 3x 1                    1 3x 1 3x
               2. u = x, dv = e3x dx, du = dx, v =        e ;    xe3x dx =      xe −         e3x dx =     xe − e + C
                                                        3                     3      3                  3     9

               3. u = x2 , dv = ex dx, du = 2x dx, v = ex ;       x2 ex dx = x2 ex − 2    xex dx.

                    For     xex dx use u = x, dv = ex dx, du = dx, v = ex to get

                         xex dx = xex − ex + C1 so     x2 ex dx = x2 ex − 2xex + 2ex + C


                                                          1                                1
               4. u = x2 , dv = e−2x dx, du = 2x dx, v = − e−2x ;            x2 e−2x dx = − x2 e−2x +         xe−2x dx
                                                          2                                2

                    For     xe−2x dx use u = x, dv = e−2x dx to get

                                     1        1                  1       1
                         xe−2x dx = − xe−2x +         e−2x dx = − xe−2x − e−2x + C
                                     2        2                  2       4
                                         1         1       1
                    so     x2 e−2x dx = − x2 e−2x − xe−2x − e−2x + C
                                         2         2       4

                                                        1
               5. u = x, dv = sin 3x dx, du = dx, v = − cos 3x;
                                                        3
                                    1           1                1          1
                    x sin 3x dx = − x cos 3x +      cos 3x dx = − x cos 3x + sin 3x + C
                                    3           3                3          9
January 27, 2005 11:45     L24-CH08         Sheet number 4 Page number 340                black



             340                                                                                                          Chapter 8


                                                            1
              6. u = x, dv = cos 2x dx, du = dx, v =          sin 2x;
                                                            2
                                     1            1                  1       1
                     x cos 2x dx =     x sin 2x −      sin 2x dx = x sin 2x + cos 2x + C
                                     2            2                  2       4

              7. u = x2 , dv = cos x dx, du = 2x dx, v = sin x;           x2 cos x dx = x2 sin x − 2      x sin x dx

                   For   x sin x dx use u = x, dv = sin x dx to get

                     x sin x dx = −x cos x + sin x + C1 so        x2 cos x dx = x2 sin x + 2x cos x − 2 sin x + C


              8. u = x2 , dv = sin x dx, du = 2x dx, v = − cos x;

                     x2 sin x dx = −x2 cos x + 2       x cos x dx; for     x cos x dx use u = x, dv = cos x dx to get


                     x cos x dx = x sin x + cos x + C1 so       x2 sin x dx = −x2 cos x + 2x sin x + 2 cos x + C

                                               1        1                          1 2        1              1 2       1
              9. u = ln x, dv = x dx, du =       dx, v = x2 ;        x ln x dx =     x ln x −     x dx =       x ln x − x2 + C
                                               x        2                          2          2              2         4

                                  √         1          2
             10. u = ln x, dv =               dx, v = x3/2 ;
                                      x dx, du =
                                            x          3
                     √            2 3/2     2             2           4
                       x ln x dx = x ln x −      x1/2 dx = x3/2 ln x − x3/2 + C
                                  3         3             3           9

                                                   ln x
             11. u = (ln x)2 , dv = dx, du = 2          dx, v = x;    (ln x)2 dx = x(ln x)2 − 2    ln x dx.
                                                    x

                   Use u = ln x, dv = dx to get       ln x dx = x ln x −        dx = x ln x − x + C1 so

                     (ln x)2 dx = x(ln x)2 − 2x ln x + 2x + C

                                 1        1         √                    ln x    √                 1      √        √
             12. u = ln x, dv = √ dx, du = dx, v = 2 x;                  √ dx = 2 x ln x−2        √ dx = 2 x ln x−4 x+C
                                  x       x                                x                        x

                                                        3                                                            3x
             13. u = ln(3x − 2), dv = dx, du =               dx, v = x;      ln(3x − 2)dx = x ln(3x − 2) −                dx
                                                      3x − 2                                                       3x − 2
                            3x                       2                   2
                   but           dx =       1+              dx = x +       ln(3x − 2) + C1 so
                          3x − 2                   3x − 2                3
                                                            2
                     ln(3x − 2)dx = x ln(3x − 2) − x −        ln(3x − 2) + C
                                                            3

                                                        2x                                                               x2
             14. u = ln(x2 + 4), dv = dx, du =         2+4
                                                           dx, v = x;        ln(x2 + 4)dx = x ln(x2 + 4) − 2                  dx
                                                      x                                                                x2 + 4
                            x2                        4                          x
                   but          dx =        1−              dx = x − 2 tan−1       + C1 so
                          x2 +4                    x2 + 4                        2
                                                                         x
                     ln(x2 + 4)dx = x ln(x2 + 4) − 2x + 4 tan−1            +C
                                                                         2
January 27, 2005 11:45       L24-CH08         Sheet number 5 Page number 341                black



              Exercise Set 8.2                                                                                                   341

                                               √
              15. u = sin−1 x, dv = dx, du = 1/ 1 − x2 dx, v = x;

                        sin−1 x dx = x sin−1 x −      x/ 1 − x2 dx = x sin−1 x +          1 − x2 + C

                                                             2
              16. u = cos−1 (2x), dv = dx, du = − √                dx, v = x;
                                                           1 − 4x2
                                                                2x                        1
                        cos−1 (2x)dx = x cos−1 (2x) +       √         dx = x cos−1 (2x) −           1 − 4x2 + C
                                                              1 − 4x2                     2

                                                       3
              17. u = tan−1 (3x), dv = dx, du =             dx, v = x;
                                                    1 + 9x2
                                                           3x                       1
                        tan−1 (3x)dx = x tan−1 (3x) −            dx = x tan−1 (3x) − ln(1 + 9x2 ) + C
                                                         1 + 9x2                    6

                                                   1           1                  1             1                            x2
              18. u = tan−1 x, dv = x dx, du =         dx, v = x2 ; x tan−1 x dx = x2 tan−1 x −                                   dx
                                                1 + x2         2                  2             2                          1 + x2
                             x2                 1
                    but           dx =   1−            dx = x − tan−1 x + C1 so
                           1 + x2            1 + x2
                                      1            1     1
                      x tan−1 x dx = x2 tan−1 x − x + tan−1 x + C
                                      2            2     2

              19. u = ex , dv = sin x dx, du = ex dx, v = − cos x;            ex sin x dx = −ex cos x +     ex cos x dx.

                    For     ex cos x dx use u = ex , dv = cos x dx to get          ex cos x = ex sin x −   ex sin x dx so

                        ex sin x dx = −ex cos x + ex sin x −        ex sin x dx,

                                                                                      1 x
                    2     ex sin x dx = ex (sin x − cos x) + C1 ,     ex sin x dx =     e (sin x − cos x) + C
                                                                                      2

                                                                      1
              20. u = e3x , dv = cos 2x dx, du = 3e3x dx, v =           sin 2x;
                                                                      2
                                       1 3x          3
                        e3x cos 2x dx =  e sin 2x −       e3x sin 2x dx. Use u = e3x , dv = sin 2x dx to get
                                       2             2
                                         1             3
                       e3x sin 2x dx = − e3x cos 2x +       e3x cos 2x dx, so
                                         2             2
                                       1             3               9
                       e3x cos 2x dx = e3x sin 2x + e3x cos 2x −          e3x cos 2x dx,
                                       2             4               4
                    13                     1                                                 1 3x
                          e3x cos 2x dx = e3x (2 sin 2x + 3 cos 2x) + C1 , e3x cos 2x dx =     e (2 sin 2x + 3 cos 2x) + C
                    4                      4                                                13

                                                                   1
              21. u = eax , dv = sin bx dx, du = aeax dx, v = − cos bx (b = 0);
                                                                   b
                                        1 ax           a
                    eax sin bx dx = − e cos bx +            eax cos bx dx. Use u = eax , dv = cos bx dx to get
                                        b              b
                                      1              a
                    eax cos bx dx = eax sin bx −         eax sin bx dx so
                                      b              b
                                        1              a               a2
                    eax sin bx dx = − eax cos bx + 2 eax sin bx − 2         eax sin bx dx,
                                        b              b               b
                                        eax
                    eax sin bx dx = 2        (a sin bx − b cos bx) + C
                                     a + b2
January 27, 2005 11:45            L24-CH08        Sheet number 6 Page number 342                        black



             342                                                                                                                   Chapter 8


                                                                      e−3θ
             22. From Exercise 21 with a = −3, b = 5, x = θ, answer = √ (−3 sin 5θ − 5 cos 5θ) + C
                                                                        34

                                                            cos(ln x)
             23. u = sin(ln x), dv = dx, du =                         dx, v = x;
                                                                x

                       sin(ln x)dx = x sin(ln x) −                  cos(ln x)dx. Use u = cos(ln x), dv = dx to get

                       cos(ln x)dx = x cos(ln x) +                  sin(ln x)dx so

                       sin(ln x)dx = x sin(ln x) − x cos(ln x) −                   sin(ln x)dx,

                                          1
                       sin(ln x)dx =        x[sin(ln x) − cos(ln x)] + C
                                          2

                                               1
             24. u = cos(ln x), dv = dx, du = − sin(ln x)dx, v = x;
                                               x

                       cos(ln x)dx = x cos(ln x) +                  sin(ln x)dx. Use u = sin(ln x), dv = dx to get

                       sin(ln x)dx = x sin(ln x) −                  cos(ln x)dx so

                       cos(ln x)dx = x cos(ln x) + x sin(ln x) −                   cos(ln x)dx,

                                          1
                       cos(ln x)dx =        x[cos(ln x) + sin(ln x)] + C
                                          2

             25. u = x, dv = sec2 x dx, du = dx, v = tan x;
                                                                                            sin x
                       x sec2 x dx = x tan x −             tan x dx = x tan x −                   dx = x tan x + ln | cos x| + C
                                                                                            cos x

             26. u = x, dv = tan2 x dx = (sec2 x − 1)dx, du = dx, v = tan x − x;

                       x tan2 x dx = x tan x − x2 −                  (tan x − x)dx

                                                                     1                  1
                                       = x tan x − x2 + ln | cos x| + x2 + C = x tan x − x2 + ln | cos x| + C
                                                                     2                  2

                                          2                                1 x2
             27. u = x2 , dv = xex dx, du = 2x dx, v =                       e ;
                                                                           2
                              2        1 2 x2                  2        1 2 x2 1 x2
                       x3 ex dx =        x e −            xex dx =        x e − e +C
                                       2                                2      2

                                           1                                   1
             28. u = xex , dv =                  dx, du = (x + 1)ex dx, v = −     ;
                                        (x + 1)2                              x+1
                         xex           xex                                   xex             ex
                                dx = −     +                   ex dx = −         + ex + C =     +C
                       (x + 1)2        x+1                                   x+1            x+1

                                                                     1 2x
             29. u = x, dv = e2x dx, du = dx, v =                      e ;
                                                                     2
                       2                      2                2                        2
                                       1 2x           1                         1                 1
                           xe2x dx =     xe       −                e2x dx = e4 − e2x        = e4 − (e4 − 1) = (3e4 + 1)/4
                   0                   2      0       2    0                    4       0         4
January 27, 2005 11:45          L24-CH08               Sheet number 7 Page number 343                                           black



              Exercise Set 8.2                                                                                                                                 343


                                                     1
              30. u = x, dv = e−5x dx, du = dx, v = − e−5x ;
                                                     5
                         1                                  1                    1
                                         1                          1
                             xe−5x dx = − xe−5x                 +                    e−5x dx
                     0                   5                  0       5        0
                                                                                1
                                             1      1                                   1     1
                                          = − e−5 − e−5x                             = − e−5 − (e−5 − 1) = (1 − 6e−5 )/25
                                             5     25                           0       5     25

                                                                1        1
              31. u = ln x, dv = x2 dx, du =                      dx, v = x3 ;
                                                                x        3
                         e                                  e                   e                                     e
                                             1 3                    1                                 1 3 1 3                 1 3 1 3
                             x2 ln x dx =      x ln x           −                    x2 dx =            e − x             =     e − (e − 1) = (2e3 + 1)/9
                     1                       3              1       3       1                         3    9          1       3    9

                                            1          1         1
              32. u = ln x, dv =             2
                                               dx, du = dx, v = − ;
                                            x          x         x
                         e                             e            e
                             ln x       1                                   1
                     √         2
                                  dx = − ln x          √
                                                            +       √
                                                                               dx
                      e       x         x               e            e      x2
                                                                                                         √
                                         1  1   √    1
                                                                                     e
                                                                                            1  1  1 1   3 e−4
                                      = − + √ ln e −                                 √
                                                                                          =− + √ − +√ =
                                         e   e       x                                e     e 2 e e  e    2e

                                                                     1
              33. u = ln(x + 2), dv = dx, du =                          dx, v = x;
                                                                    x+2
                         1                                              1                 1                                         1
                                                                                               x                                              2
                             ln(x + 2)dx = x ln(x + 2)                       −                    dx = ln 3 + ln 1 −                    1−       dx
                     −1                                                 −1               −1   x+2                                  −1        x+2
                                                                                                  1
                                             = ln 3 − [x − 2 ln(x + 2)]                                 = ln 3 − (1 − 2 ln 3) + (−1 − 2 ln 1) = 3 ln 3 − 2
                                                                                                  −1

                                                                      1
              34. u = sin−1 x, dv = dx, du = √                             dx, v = x;
                                                                    1 − x2
                         √
                          3/2
                                                                    √
                                                                     3/2
                                                                                              √
                                                                                               3/2
                                                                                                                     √         √                        √
                                                                                                                                                         3/2
                                     −1                    −1                                              x           3         3
                                sin       x dx = x sin          x                   −                  √        dx =     sin−1     +           1 − x2
                     0                                              0                     0              1 − x2       2         2                       0
                                                   √                                              √
                                                    3       π  1    π 3 1
                                               =              + −1=    −
                                                   2        3  2     6   2
                                      √                                1
              35. u = sec−1               θ, dv = dθ, du =            √      dθ, v = θ;
                                                                    2θ θ − 1
                         4           √                      √1
                                                                    4
                                                                                1       √ 4 √                                                           4
                             sec−1       θdθ = θ sec−1          dθ = 4 sec−1 2 − 2 sec−1 2 − θ − 1
                                                                θ       −                     √
                     2                               2    2 θ−1                 2                                                                       2

                                                π     π  √     5π √
                                             =4   −2    − 3+1=     − 3+1
                                                3     4         6
                                                 1          1
              36. u = sec−1 x, dv = x dx, du = √     dx, v = x2 ;
                                              x x2−1        2
                         2                                              2                     2
                                                 1 2                                 1                 x
                             x sec−1 x dx =        x sec−1 x                −                     √          dx
                     1                           2                      1            2    1           x2 − 1
                                                 1                       1
                                                                                                                  2                √
                                             =     [(4)(π/3) − (1)(0)] −                                x2 − 1        = 2π/3 −         3/2
                                                 2                       2                                        1
January 27, 2005 11:45            L24-CH08               Sheet number 8 Page number 344                                                   black



             344                                                                                                                                                         Chapter 8


                                                      1
             37. u = x, dv = sin 2x dx, du = dx, v = − cos 2x;
                                                      2
                        π                                          π                    π                                                          π
                                           1                                  1                                                   1
                            x sin 2x dx = − x cos 2x                      +                 cos 2x dx = −π/2 +                      sin 2x             = −π/2
                    0                      2                       0          2     0                                             4                0

                        π                                     π               π                                            π
                                                      1 2                                                 π2
             38.            (x + x cos x)dx =           x         +               x cos x dx =               +                 x cos x dx;
                    0                                 2       0           0                               2            0

                   u = x, dv = cos x dx, du = dx, v = sin x
                        π                                π            π                                       π                       π
                            x cos x dx = x sin x             −            sin x dx = cos x                        = −2 so                 (x + x cos x)dx = π 2 /2 − 2
                    0                                    0        0                                           0                   0

                                      √             √                1             2
             39. u = tan−1                x, dv =       xdx, du = √         dx, v = x3/2 ;
                                                                 2 x(1 + x)        3
                        3   √             √            2 3/2    √                       3
                                                                                                 1        3
                                                                                                               x
                                x tan−1       xdx =      x tan−1 x                          −                     dx
                    1                                  3                                1        3    1       1+x
                                                       2 3/2    √                       3
                                                                                                 1        3
                                                                                                                        1
                                                  =      x tan−1 x                          −                     1−       dx
                                                       3                                1        3    1                1+x
                                                        2 3/2    √   1   1
                                                                                                                                  3       √
                                                  =       x tan−1 x − x + ln |1 + x|                                                  = (2 3π − π/2 − 2 + ln 2)/3
                                                        3            3   3                                                        1

                                                                         2x
             40. u = ln(x2 + 1), dv = dx, du =                                dx, v = x;
                                                                       x2 + 1
                        2                                                 2             2                                                     2
                                                                                             2x2                                                          1
                            ln(x2 + 1)dx = x ln(x2 + 1)                       −                  dx = 2 ln 5 − 2                                  1−            dx
                    0                                                     0         0       x2+1                                          0            x2 + 1
                                                                                                 2
                                               = 2 ln 5 − 2(x − tan−1 x)                             = 2 ln 5 − 4 + 2 tan−1 2
                                                                                                 0
                            √
             41. t =            x, t2 = x, dx = 2t dt
                                   √
                                    x
                   (a)            e     dx = 2        tet dt; u = t, dv = et dt, du = dt, v = et ,

                                   √                                                         √       √
                                  e x
                                        dx = 2tet − 2             et dt = 2(t − 1)et + C = 2( x − 1)e x + C

                                        √
                   (b)            cos       x dx = 2         t cos t dt; u = t, dv = cos tdt, du = dt, v = sin t,

                                        √                                                                             √     √         √
                                  cos       x dx = 2t sin t − 2                   sin tdt = 2t sin t + 2 cos t + C = 2 x sin x + 2 cos x + C
January 27, 2005 11:45      L24-CH08             Sheet number 9 Page number 345         black



              Exercise Set 8.2                                                                                          345


              42. Let q1 (x), q2 (x), q3 (x) denote successive antiderivatives of q(x),
                  so that q3 (x) = q2 (x), q2 (x) = q1 (x), q1 (x) = q(x). Let p(x) = ax2 + bx + c.

                                     Repeated                Repeated
                                   Differentiation        Antidifferentiation
                                    ax2 + bx + c                 q(x)
                                                         +
                                       2ax + b                  q1 (x)
                                                         −
                                         2a                     q2 (x)
                                                         +
                                          0                     q3 (x)

                    Then         p(x)q(x) dx = (ax2 + bx + c)q1 (x) − (2ax + b)q2 (x) + 2aq3 (x) + C. Check:

                     d
                       [(ax2 +bx + c)q1 (x) − (2ax + b)q2 (x) + 2aq3 (x)]
                    dx
                                 = (2ax + b)q1 (x) + (ax2 + bx + c)q(x) − 2aq2 (x) − (2ax + b)q1 (x) + 2aq2 (x) = p(x)q(x)

              43.                    Repeated                Repeated
                                   Differentiation        Antidifferentiation
                                    3x2 − x + 2                 e−x
                                                       +
                                       6x − 1                 −e−x
                                                       −
                                         6                      e−x
                                                       +
                                         0                    −e−x

                      (3x2 − x + 2)e−x = −(3x2 − x + 2)e−x − (6x − 1)e−x − 6e−x + C = −e−x [3x2 + 5x + 7] + C


              44.                    Repeated                Repeated
                                   Differentiation        Antidifferentiation
                                    x2 + x + 1                 sin x
                                                     +
                                      2x + 1                 − cos x
                                                     −
                                         2                   − sin x
                                                     +
                                         0                     cos x

                      (x2 + x + 1) sin x dx = −(x2 + x + 1) cos x + (2x + 1) sin x + 2 cos x + C
                                                 = −(x2 + x − 1) cos x + (2x + 1) sin x + C
January 27, 2005 11:45           L24-CH08                      Sheet number 10 Page number 346                       black



             346                                                                                                                                 Chapter 8


             45.                        Repeated                          Repeated
                                      Differentiation                  Antidifferentiation
                                              4x4                                  sin 2x
                                                                +
                                                                           1
                                             16x3                         − cos 2x
                                                                           2
                                                                −
                                                                           1
                                             48x       2
                                                                          − sin 2x
                                                                +          4
                                                                              1
                                              96x                               cos 2x
                                                                              8
                                                                −
                                                                              1
                                               96                                sin 2x
                                                                              16
                                                                +
                                                                              1
                                               0                         −      cos 2x
                                                                             32

                     4x4 sin 2x dx = (−2x4 + 6x2 − 3) cos 2x + −(4x3 + 6x) sin 2x + C


             46.                        Repeated                         Repeated
                                      Differentiation                 Antidifferentiation
                                                                          √
                                              x3                            2x + 1
                                                                +
                                                                         1
                                             3x    2
                                                                           (2x + 1)3/2
                                                                         3
                                                                −
                                                                        1
                                              6x                           (2x + 1)5/2
                                                                        15
                                                                +
                                                                         1
                                               6                            (2x + 1)7/2
                                                                        105
                                                                −
                                                                         1
                                               0                            (2x + 1)9/2
                                                                        945
                       √           1                1                2                2
                     x3 2x + 1 dx = x3 (2x + 1)3/2 − x2 (2x + 1)5/2 + x(2x + 1)7/2 −     (2x + 1)9/2 + C
                                   3                5                35              315

             47. (a) We perform a single integration by parts:
                         u = cos x, dv = sin x dx, du = − sin x dx, v = − cos x,

                                 sin x cos x dx = − cos2 x −                       sin x cos x dx. Thus

                                                                                                         1
                         2           sin x cos x dx = − cos2 x + C,                    sin x cos x dx = − cos2 x + C
                                                                                                         2
                                                                                                              1 2      1
                   (b) u = sin x, du = cos x dx,                         sin x cos x dx =            u du =     u + C = sin2 x + C
                                                                                                              2        2
                                                           x
             48. (a) u = x2 , dv = √                                , du = 2x dx, v =            x2 + 1,
                                                       x2      +1
                                 1
                                          x3
                                                                               1           1                    √      2
                                                                                                                                      1
                                                                                                                                               1√    2
                                     √          dx = x2              x2 + 1        −           2x x2 + 1 dx =       2 − (x2 + 1)3/2       =−      2+
                             0           x2 + 1                                0       0                               3              0        3     3
January 27, 2005 11:45             L24-CH08                             Sheet number 11 Page number 347                                        black



              Exercise Set 8.2                                                                                                                                                          347

                                                                                                           √                                                √
                                                                                                            2                                                2
                                                                                   x                                                     1 3
                    (b) u =                  x2          + 1, du = √                         dx,                 (u − 1) du =
                                                                                                                     2
                                                                                                                                           u −u
                                                                              x2 + 1                   1                                 3                  1
                                2√   √  1     1√   2
                              =    2− 2− +1=−    2+ .
                                3       3     3    3
                                                 e                                            e
              49. (a) A =                            ln x dx = (x ln x − x)                        =1
                                             1                                                1
                                                         e                                                                      e
                    (b) V = π                                (ln x)2 dx = π (x(ln x)2 − 2x ln x + 2x)                               = π(e − 2)
                                                     1                                                                          1


                                   π/2                                                   π/2               π/2                                                          π/2
                                                                               1 2                                                  π2
              50. A =                    (x − x sin x)dx =                       x              −                x sin x dx =          − (−x cos x + sin x)                   = π 2 /8 − 1
                               0                                               2         0             0                            8                                   0

                                         π                                                                       π
              51. V = 2π                     x sin x dx = 2π(−x cos x + sin x)                                       = 2π 2
                                     0                                                                           0

                                         π/2                                                                     π/2
              52. V = 2π                             x cos x dx = 2π(cos x + x sin x)                                    = π(π − 2)
                                     0                                                                           0

                                                     π
              53. distance =                             t3 sin tdt;
                                                 0

                                           Repeated                                    Repeated
                                         Differentiation                            Antidifferentiation
                                                              t3                        sin t
                                                                          +
                                                              3t2                      − cos t
                                                                          −
                                                              6t                       − sin t
                                                                          +
                                                               6                        cos t
                                                                          −
                                                               0                        sin t

                         π                                                                                                          π
                             t3 sin t dx = [(−t3 cos t + 3t2 sin t + 6t cos t − 6 sin t)]                                               = π 3 − 6π
                     0                                                                                                              0

                                                                                                        1
              54. u = 2t, dv = sin(kωt)dt, du = 2dt, v = −                                                cos(kωt); the integrand is an even function of t so
                                                                                                       kω
                         π/ω                                                 π/ω                                                         π/ω               π/ω
                                                                                                                      2                                           1
                               t sin(kωt) dt = 2                                   t sin(kωt) dt = −                    t cos(kωt)             +2                   cos(kωt) dt
                     −π/ω                                                0                                           kω                  0             0         kω
                                                                                   k+1                                    π/ω                   k+1
                                                                       2π(−1)                      2                                2π(−1)
                                                                   =                     +                 sin(kωt)             =
                                                                          kω 2                k2 ω2                       0            kω 2

                                                 1               3
              55. (a)               sin4 x dx = − sin3 x cos x +                                           sin2 x dx
                                                 4               4
                                                                 1               3  1             1
                                                              = − sin3 x cos x +   − sin x cos x + x + C
                                                                 4               4  2             2
                                                                 1              3             3
                                                              = − sin3 x cos x − sin x cos x + x + C
                                                                 4              8             8
January 27, 2005 11:45         L24-CH08            Sheet number 12 Page number 348                              black



             348                                                                                                                   Chapter 8


                               π/2                                   π/2                 π/2
                                                  1                             4
                   (b)               sin5 x dx = − sin4 x cos x            +                   sin3 x dx
                           0                      5                  0          5    0
                                                                           π/2                 π/2
                                                   4  1                               2
                                               =     − sin2 x cos x              +                   sin x dx
                                                   5  3                    0          3    0
                                                               π/2
                                                     8                    8
                                               =−      cos x         =
                                                    15         0         15

                                               1                4                              1                4 1               2
             56. (a)           cos5 x dx =       cos4 x sin x +          cos3 x dx =             cos4 x sin x +     cos2 x sin x + sin x + C
                                               5                5                              5                5 3               3
                                               1                4                 8
                                           =     cos4 x sin x +    cos2 x sin x +    sin x + C
                                               5                15                15
                                               1                5
                   (b)         cos6 x dx =       cos5 x sin x +          cos4 x dx
                                               6                6
                                               1                5 1                3
                                           =     cos5 x sin x +     cos3 x sin x +                      cos2 x dx
                                               6                6 4                4
                                               1                5                 5 1              1
                                           =     cos5 x sin x +    cos3 x sin x +     cos x sin x + x + C,
                                               6                24                8 2              2
                                                                                                         π/2
                            1                5                 5                5
                              cos5 x sin x +    cos3 x sin x +    cos x sin x + x                               = 5π/32
                            6                24                16              16                        0


             57. u = sinn−1 x, dv = sin x dx, du = (n − 1) sinn−2 x cos x dx, v = − cos x;

                       sinn x dx = − sinn−1 x cos x + (n − 1)                 sinn−2 x cos2 x dx

                                     = − sinn−1 x cos x + (n − 1)             sinn−2 x (1 − sin2 x)dx

                                     = − sinn−1 x cos x + (n − 1)             sinn−2 x dx − (n − 1)               sinn x dx,


                   n      sinn x dx = − sinn−1 x cos x + (n − 1)                    sinn−2 x dx,

                                           1                  n−1
                         sinn x dx = −       sinn−1 x cos x +                   sinn−2 x dx
                                           n                   n

             58. (a) u = secn−2 x, dv = sec2 x dx, du = (n − 2) secn−2 x tan x dx, v = tan x;

                               secn x dx = secn−2 x tan x − (n − 2)                 secn−2 x tan2 x dx

                                           = secn−2 x tan x − (n − 2)               secn−2 x (sec2 x − 1)dx

                                           = secn−2 x tan x − (n − 2)               secn x dx + (n − 2)             secn−2 x dx,


                          (n − 1)        secn x dx = secn−2 x tan x + (n − 2)                    secn−2 x dx,

                                                1                   n−2
                               secn x dx =         secn−2 x tan x +                            secn−2 x dx
                                               n−1                  n−1
January 27, 2005 11:45         L24-CH08                   Sheet number 13 Page number 349                        black



              Exercise Set 8.2                                                                                                                     349


                    (b)            tann x dx =            tann−2 x (sec2 x − 1) dx =               tann−1 x sec2 x dx −        tann−2 x dx

                                                        1
                                                   =       tann−1 x −               tann−2 x dx
                                                       n−1

                    (c) u = xn , dv = ex dx, du = nxn−1 dx, v = ex ;                               xn ex dx = xn ex − n       xn−1 ex dx

                                                       1                                    1                                 1
              59. (a)              tan4 x dx =           tan3 x −             tan2 x dx =     tan3 x − tan x +         dx =     tan3 x − tan x + x + C
                                                       3                                    3                                 3
                                                       1                2                           1               2
                    (b)            sec4 x dx =           sec2 x tan x +             sec2 x dx =       sec2 x tan x + tan x + C
                                                       3                3                           3               3

                    (c)            x3 ex dx = x3 ex − 3                 x2 ex dx = x3 ex − 3 x2 ex − 2        xex dx


                                                 = x3 ex − 3x2 ex + 6 xex −                 ex dx = x3 ex − 3x2 ex + 6xex − 6ex + C

              60. (a) u = 3x,
                                                      1                         1                              1 2 u   2
                                   x2 e3x dx =              u2 eu du =            u2 eu − 2        ueu du =      u e −    ueu −            eu du
                                                     27                        27                             27       27
                                                      1 2 u  2     2        1        2      2
                                                 =      u e − ueu + eu + C = x2 e3x − xe3x + e3x + C
                                                     27      27    27       3        9      27
                             √
                    (b) u = − x,
                                   1        √                  −1
                                       xe−   x
                                                 dx = 2             u3 eu du,
                               0                           0


                                   u3 eu du = u3 eu − 3                 u2 eu du = u3 eu − 3 u2 eu − 2        ueu du


                                                 = u3 eu − 3u2 eu + 6 ueu −                 eu du = u3 eu − 3u2 eu + 6ueu − 6eu + C,

                                       −1                                                    −1
                           2                u3 eu du = 2(u3 − 3u2 + 6u − 6)eu                     = 12 − 32e−1
                                   0                                                         0


              61. u = x, dv = f (x)dx, du = dx, v = f (x);
                      1                                    1              1
                          x f (x)dx = xf (x)                        −         f (x)dx
                     −1                                    −1            −1

                                                                                    1
                                              = f (1) + f (−1) − f (x)                   = f (1) + f (−1) − f (1) + f (−1)
                                                                                    −1


              62. (a)              u dv = uv −            v du = x(sin x + C1 ) + cos x − C1 x + C2 = x sin x + cos x + C2 ;

                           the constant C1 cancels out and hence plays no role in the answer.

                    (b) u(v + C1 ) −                   (v + C1 )du = uv + C1 u −                 v du − C1 u = uv −       v du
January 27, 2005 11:45        L24-CH08             Sheet number 14 Page number 350                                        black



             350                                                                                                                                                  Chapter 8


                                                                dx
             63. u = ln(x + 1), dv = dx, du =                      , v = x + 1;
                                                               x+1
                     ln(x + 1) dx =           u dv = uv −                   v du = (x + 1) ln(x + 1) −                     dx = (x + 1) ln(x + 1) − x + C


                                                                   3dx           2
             64. u = ln(3x − 2), dv = dx, du =                           ,v = x − ;
                                                                  3x − 2         3
                                                                                                       2                                  2          1
                     ln(3x − 2) dx =            u dv = uv −                  v du =              x−           ln(3x − 2) −           x−                   dx
                                                                                                       3                                  3       x − 2/3
                                                    2                                              2
                                         =    x−              ln(3x − 2) − x −                             +C
                                                    3                                              3

                                                         1            1
             65. u = tan−1 x, dv = x dx, du =              2
                                                             dx, v = (x2 + 1)
                                                       1+x            2
                                                                    1 2            1
                     x tan−1 x dx =           u dv = uv − v du = (x + 1) tan−1 x −                                                 dx
                                                                    2              2
                                            1 2                1
                                        =     (x + 1) tan−1 x − x + C
                                            2                  2

                         1
             66. u =        , dv = x dx, du = − x(ln x)2 dx,
                                      1            1
                                                                                                  v = ln x
                       ln x
                        1                  1
                             dx = 1 +          dx.
                     x ln x             x ln x
                   This seems to imply that 1 = 0, but recall that both sides represent a function plus an arbitrary
                   constant; these two arbitrary constants will take care of the 1.

             67. (a) u = f (x), dv = dx, du = f (x), v = x;
                              b                           b            b                                                      b
                                  f (x) dx = xf (x)           −            xf (x) dx = bf (b) − af (a) −                          xf (x) dx
                          a                               a        a                                                      a

                   (b) Substitute y = f (x), dy = f (x) dx, x = a when y = f (a), x = b when y = f (b),
                              b                    f (b)                      f (b)
                                  xf (x) dx =                 x dy =                  f −1 (y) dy
                          a                       f (a)                      f (a)
                                                                                                                                              y
                   (c) From a = f −1 (α) and b = f −1 (β) we get                                                                         b
                         bf (b) − af (a) = βf −1 (β) − αf −1 (α); then                                                                                  A1
                              β                     β                                    f (b)
                                  f −1 (x) dx =         f −1 (y) dy =                            f −1 (y) dy,                            a                   A2
                          α                        α                                  f (a)                                                                               x
                         which, by Part (b), yields                                                                                       a=      f –1(a)         b=f   –1(b)


                              β                                                          b
                                  f −1 (x) dx = bf (b) − af (a) −                            f (x) dx
                          α                                                          a
                                                                                                   f −1 (β)
                                                       −1                    −1
                                             = βf             (β) − αf            (α) −                       f (x) dx
                                                                                                  f −1 (α)
                                                                                     β                               f −1 (β)
                         Note from the figure that A1 =                                   f −1 (x) dx, A2 =                        f (x) dx, and
                                                                                  α                                 f −1 (α)

                         A1 + A2 = βf −1 (β) − αf −1 (α), a “picture proof”.
January 27, 2005 11:45      L24-CH08                       Sheet number 15 Page number 351                       black



              Exercise Set 8.3                                                                                                                               351


              68. (a) Use Exercise 67(c);
                                1/2                                                             sin−1 (1/2)                                      π/6
                                                             1         1                                                     1         1
                                      sin−1 x dx =             sin−1     −0·sin−1 0−                          sin x dx =       sin−1     −                sin x dx
                            0                                2         2                       sin−1 (0)                     2         2     0


                    (b) Use Exercise 67(b);
                                e2                                              ln e2                                   2                             2
                                     ln x dx = e2 ln e2 − e ln e −                      f −1 (y) dy = 2e2 − e −             ey dy = 2e2 − e −             ex dx
                            e                                               ln e                                    1                             1




              EXERCISE SET 8.3
                                                        1
               1. u = cos x, −                 u3 du = − cos4 x + C
                                                        4

                                      1                       1
               2. u = sin 3x,                  u5 du =          sin6 3x + C
                                      3                      18

                                       1                                 1    1
               3.      sin2 5θ =                (1 − cos 10θ) dθ =         θ−    sin 10θ + C
                                       2                                 2    20

                                               1                            1     1
               4.      cos2 3x dx =                    (1 + cos 6x)dx =       x+    sin 6x + C
                                               2                            2    12

                                                                             1          1
               5.      sin3 aθ dθ =                sin aθ(1 − cos2 aθ) dθ = − cos aθ −    cos3 aθ + C                        (a = 0)
                                                                             a         3a

               6.      cos3 at dt =             (1 − sin2 at) cos at dt

                                                                                             1           1
                                       =           cos at dt −     sin2 at cos at dt =         sin at −    sin3 at + C (a = 0)
                                                                                             a          3a

                                           1                   1
               7. u = sin ax,                       u du =       sin2 ax + C, a = 0
                                           a                  2a

               8.      sin3 x cos3 x dx =                  sin3 x(1 − sin2 x) cos x dx

                                                                                              1         1
                                                   =       (sin3 x − sin5 x) cos x dx =         sin4 x − sin6 x + C
                                                                                              4         6

               9.      sin2 t cos3 t dt =               sin2 t(1 − sin2 t) cos t dt =         (sin2 t − sin4 t) cos t dt

                                                    1         1
                                               =      sin3 t − sin5 t + C
                                                    3         5

              10.      sin3 x cos2 x dx =                  (1 − cos2 x) cos2 x sin x dx

                                                                                         1        1
                                                   =       (cos2 x − cos4 x) sin x dx = − cos3 x + cos5 x + C
                                                                                         3        5

                                                       1                    1                              1     1
              11.      sin2 x cos2 x dx =                    sin2 2x dx =           (1 − cos 4x)dx =         x−    sin 4x + C
                                                       4                    8                              8    32
January 27, 2005 11:45           L24-CH08                    Sheet number 16 Page number 352                                        black



             352                                                                                                                                                  Chapter 8


                                                    1                                                                1
             12.        sin2 x cos4 x dx =                    (1 − cos 2x)(1 + cos 2x)2 dx =                                (1 − cos2 2x)(1 + cos 2x)dx
                                                    8                                                                8
                                                    1                                   1                                       1                           1
                                            =                 sin2 2x dx +                    sin2 2x cos 2x dx =                        (1 − cos 4x)dx +      sin3 2x
                                                    8                                   8                                      16                           48
                                                     1    1            1
                                            =          x−    sin 4x +    sin3 2x + C
                                                    16    64          48
                                                     1                                                 1          1
             13.        sin 2x cos 3x dx =                       (sin 5x − sin x)dx = −                   cos 5x + cos x + C
                                                     2                                                 10         2

                                                    1                                                 1          1
             14.        sin 3θ cos 2θdθ =                     (sin 5θ + sin θ)dθ = −                     cos 5θ − cos θ + C
                                                    2                                                 10         2

                                                         1                                    1
             15.        sin x cos(x/2)dx =                        [sin(3x/2) + sin(x/2)]dx = − cos(3x/2) − cos(x/2) + C
                                                         2                                    3

                                                   3
             16. u = cos x, −           u1/3 du = − cos4/3 x + C
                                                   4

                        π/2                         π/2
             17.              cos3 x dx =                   (1 − sin2 x) cos x dx
                    0                           0
                                                                                    π/2
                                                                  1                           2
                                       = sin x −                    sin3 x                =
                                                                  3                 0         3

                        π/2                                                         π/2                              π/2
                                                                            1                               1
             18.              sin2 (x/2) cos2 (x/2)dx =                                   sin2 x dx =                      (1 − cos 2x)dx
                    0                                                       4   0                           8    0
                                                                                                           π/2
                                                                            1             1
                                                                      =         x−          sin 2x               = π/16
                                                                            8             2                0

                        π/3                                           π/3                                                                                     π/3
                                                                                                                                   1             1
             19.             sin4 3x cos3 3x dx =                         sin4 3x(1 − sin2 3x) cos 3x dx =                            sin5 3x −    sin7 3x        =0
                    0                                             0                                                                15           21            0

                        π                               π                                                                      π
                                            1                                                 1            1
             20.             cos2 5θ dθ =                   (1 + cos 10θ)dθ =                         θ+      sin 10θ                   =π
                    −π                      2       −π                                        2            10                  −π

                        π/6                                           π/6                                                                         π/6
                                                             1                                     1          1
             21.              sin 4x cos 2x dx =                            (sin 2x + sin 6x)dx = − cos 2x −    cos 6x
                    0                                        2    0                                4         12                                   0
                                                     = [(−1/4)(1/2) − (1/12)(−1)] − [−1/4 − 1/12] = 7/24

                        2π                               2π                                                                         2π
                                            1                                                     1             1                               1
             22.             sin2 kx dx =                     (1 − cos 2kx)dx =                       x−          sin 2kx                =π−      sin 4πk (k = 0)
                    0                       2        0                                            2            2k                   0          4k

                   1                                                                                       1
             23.     tan(2x − 1) + C                                                                  24. − ln | cos 5x| + C
                   2                                                                                       5

             25. u = e−x , du = −e−x dx; −                              tan u du = ln | cos u| + C = ln | cos(e−x )| + C

                   1                                                                                            1
             26.     ln | sin 3x| + C                                                                 27.         ln | sec 4x + tan 4x| + C
                   3                                                                                            4
January 27, 2005 11:45       L24-CH08              Sheet number 17 Page number 353               black



              Exercise Set 8.3                                                                                                       353


                         √           1                                                                      √          √
              28. u =        x, du = √ dx;            2 sec u du = 2 ln | sec u + tan u| + C = 2 ln sec         x + tan x + C
                                    2 x

                                                  1
              29. u = tan x,            u2 du =     tan3 x + C
                                                  3

                                                                                                 1         1
              30.      tan5 x(1 + tan2 x) sec2 x dx =            (tan5 x + tan7 x) sec2 x dx =     tan6 x + tan8 x + C
                                                                                                 6         8

                                                                                                       1            1
              31.      tan 4x(1 + tan2 4x) sec2 4x dx =            (tan 4x + tan3 4x) sec2 4x dx =       tan2 4x +    tan4 4x + C
                                                                                                       8           16

                                                             1         1
              32.      tan4 θ(1 + tan2 θ) sec2 θ dθ =          tan5 θ + tan7 θ + C
                                                             5         7

                                                                                                           1         1
              33.      sec4 x(sec2 x − 1) sec x tan x dx =           (sec6 x − sec4 x) sec x tan x dx =      sec7 x − sec5 x + C
                                                                                                           7         5

                                                                                                         1         2
              34.     (sec2 θ − 1)2 sec θ tan θdθ =          (sec4 θ − 2 sec2 θ + 1) sec θ tan θdθ =       sec5 θ − sec3 θ + sec θ + C
                                                                                                         5         3

              35.     (sec2 x − 1)2 sec x dx =          (sec5 x − 2 sec3 x + sec x)dx =      sec5 x dx − 2        sec3 x dx +    sec x dx

                             1                3
                         =     sec3 x tan x +          sec3 x dx − 2     sec3 x dx + ln | sec x + tan x|
                             4                4
                             1                5 1              1
                         =     sec3 x tan x −     sec x tan x + ln | sec x + tan x| + ln | sec x + tan x| + C
                             4                4 2              2
                             1               5             3
                         =     sec3 x tan x − sec x tan x + ln | sec x + tan x| + C
                             4               8             8

              36.     [sec2 x − 1] sec3 x dx =         [sec5 x − sec3 x]dx

                                  1                3
                           =        sec3 x tan x +        sec3 x dx −        sec3 x dx            (equation (20))
                                  4                4
                                 1                1
                           =       sec3 x tan x −        sec3 x dx
                                 4                4
                                 1               1             1
                           =       sec3 x tan x − sec x tan x − ln | sec x + tan x| + C           (equation (20), (22))
                                 4               8             8
                                                    1                                                             1
              37.      sec2 t(sec t tan t)dt =        sec3 t + C             38.      sec4 x(sec x tan x)dx =       sec5 x + C
                                                    3                                                             5

                                                                                                                   1
              39.      sec4 x dx =         (1 + tan2 x) sec2 x dx =      (sec2 x + tan2 x sec2 x)dx = tan x +        tan3 x + C
                                                                                                                   3

              40. Using equation (20),
                                        1                3
                       sec5 x dx =        sec3 x tan x +         sec3 x dx
                                        4                4
                                        1               3             3
                                    =     sec3 x tan x + sec x tan x + ln | sec x + tan x| + C
                                        4               8             8
January 27, 2005 11:45                L24-CH08               Sheet number 18 Page number 354                           black



             354                                                                                                                                Chapter 8


             41. u = 4x, use equation (19) to get
                   1                                  1 1                           1         1
                               tan3 u du =                tan2 u + ln | cos u| + C = tan2 4x + ln | cos 4x| + C
                   4                                  4 2                           8         4

                                                                                     1
             42. Use equation (19) to get                         tan4 x dx =          tan3 x − tan x + x + C
                                                                                     3
                           √                                               2           2
             43.               tan x(1 + tan2 x) sec2 x dx =                 tan3/2 x + tan7/2 x + C
                                                                           3           7

                                                                  2
             44.           sec1/2 x(sec x tan x)dx =                sec3/2 x + C
                                                                  3

                           π/8                                                  π/8
                                                                1
             45.                 (sec2 2x − 1)dx =                tan 2x − x              = 1/2 − π/8
                       0                                        2               0

                           π/6                                                        π/6
                                                                          1
             46.                 sec2 2θ(sec 2θ tan 2θ)dθ =                 sec3 2θ             = (1/6)(2)3 − (1/6)(1) = 7/6
                       0                                                  6           0

             47. u = x/2,
                               π/4
                                                          1
                                                                                                        π/4                      √
                   2                 tan5 u du =            tan4 u − tan2 u − 2 ln | cos u|                   = 1/2 − 1 − 2 ln(1/ 2) = −1/2 + ln 2
                           0                              2                                             0


                                       1        π/4
                                                                           1
                                                                                      π/4      √
             48. u = πx,                              sec u tan u du =       sec u          = ( 2 − 1)/π
                                       π    0                              π          0

                                                                                                                     1        1
             49.           (csc2 x − 1) csc2 x(csc x cot x)dx =                  (csc4 x − csc2 x)(csc x cot x)dx = − csc5 x + csc3 x + C
                                                                                                                     5        3

                           cos2 3t   1                                              1
             50.                   ·      dt =                  csc 3t cot 3t dt = − csc 3t + C
                           sin2 3t cos 3t                                           3

                                                                                                        cos x       1
             51.           (csc2 x − 1) cot x dx =                csc x(csc x cot x)dx −                      dx = − csc2 x − ln | sin x| + C
                                                                                                        sin x       2

                                                     1
             52.           (cot2 x + 1) csc2 x dx = − cot3 x − cot x + C
                                                     3
                                      2π                                  2π
                                                                  1
             53. (a)                  sin mx cos nx dx =                   [sin(m + n)x + sin(m − n)x]dx
                                  0                               2   0
                                                                                                                  2π
                                                                      cos(m + n)x cos(m − n)x
                                                                = −              −
                                                                        2(m + n)    2(m − n)                      0
                                                            2π                              2π
                                 but cos(m + n)x                 = 0, cos(m − n)x                = 0.
                                                            0                               0
                                      2π
                                                         1 2π
                   (b)                     cos mx cos nx dx =   [cos(m + n)x + cos(m − n)x]dx;
                                   0                     2 0
                                 since m = n, evaluate sin at integer multiples of 2π to get 0.
                                      2π
                                                         1 2π
                   (c)                     sin mx sin nx dx =   [cos(m − n)x − cos(m + n)x] dx;
                                   0                     2 0
                                 since m = n, evaluate sin at integer multiples of 2π to get 0.
Chapter 08
Chapter 08
Chapter 08
Chapter 08
Chapter 08
Chapter 08
Chapter 08
Chapter 08
Chapter 08
Chapter 08
Chapter 08
Chapter 08
Chapter 08
Chapter 08
Chapter 08
Chapter 08
Chapter 08
Chapter 08
Chapter 08
Chapter 08
Chapter 08
Chapter 08
Chapter 08
Chapter 08
Chapter 08
Chapter 08
Chapter 08
Chapter 08
Chapter 08
Chapter 08
Chapter 08
Chapter 08
Chapter 08
Chapter 08
Chapter 08
Chapter 08
Chapter 08
Chapter 08
Chapter 08
Chapter 08

Contenu connexe

Tendances

ตัวอย่างข้อสอบเก่า วิชาคณิตศาสตร์ ม.6 ปีการศึกษา 2553
ตัวอย่างข้อสอบเก่า วิชาคณิตศาสตร์ ม.6 ปีการศึกษา 2553ตัวอย่างข้อสอบเก่า วิชาคณิตศาสตร์ ม.6 ปีการศึกษา 2553
ตัวอย่างข้อสอบเก่า วิชาคณิตศาสตร์ ม.6 ปีการศึกษา 2553Destiny Nooppynuchy
 
Antiderivatives nako sa calculus official
Antiderivatives nako sa calculus officialAntiderivatives nako sa calculus official
Antiderivatives nako sa calculus officialZerick Lucernas
 
Diifferential equation akshay
Diifferential equation akshayDiifferential equation akshay
Diifferential equation akshayakshay1234kumar
 
Calculus First Test 2011/10/20
Calculus First Test 2011/10/20Calculus First Test 2011/10/20
Calculus First Test 2011/10/20Kuan-Lun Wang
 
Gaussseidelsor
GaussseidelsorGaussseidelsor
Gaussseidelsoruis
 
Pt 2 turunan fungsi eksponen, logaritma, implisit dan cyclometri-d4
Pt 2 turunan fungsi eksponen, logaritma, implisit dan cyclometri-d4Pt 2 turunan fungsi eksponen, logaritma, implisit dan cyclometri-d4
Pt 2 turunan fungsi eksponen, logaritma, implisit dan cyclometri-d4lecturer
 
数式処理ソフトMathematicaで数学の問題を解く
数式処理ソフトMathematicaで数学の問題を解く数式処理ソフトMathematicaで数学の問題を解く
数式処理ソフトMathematicaで数学の問題を解くYoshihiro Mizoguchi
 
Derivative of an exponential function
Derivative of an exponential functionDerivative of an exponential function
Derivative of an exponential functionNadeem Uddin
 

Tendances (20)

Chapter 15
Chapter 15Chapter 15
Chapter 15
 
Deber10
Deber10Deber10
Deber10
 
ตัวอย่างข้อสอบเก่า วิชาคณิตศาสตร์ ม.6 ปีการศึกษา 2553
ตัวอย่างข้อสอบเก่า วิชาคณิตศาสตร์ ม.6 ปีการศึกษา 2553ตัวอย่างข้อสอบเก่า วิชาคณิตศาสตร์ ม.6 ปีการศึกษา 2553
ตัวอย่างข้อสอบเก่า วิชาคณิตศาสตร์ ม.6 ปีการศึกษา 2553
 
Sect1 4
Sect1 4Sect1 4
Sect1 4
 
Antiderivatives nako sa calculus official
Antiderivatives nako sa calculus officialAntiderivatives nako sa calculus official
Antiderivatives nako sa calculus official
 
Derivadas
DerivadasDerivadas
Derivadas
 
2º mat emática
2º mat emática2º mat emática
2º mat emática
 
Sect1 5
Sect1 5Sect1 5
Sect1 5
 
Diifferential equation akshay
Diifferential equation akshayDiifferential equation akshay
Diifferential equation akshay
 
Ism et chapter_6
Ism et chapter_6Ism et chapter_6
Ism et chapter_6
 
Lesson 15: The Chain Rule
Lesson 15: The Chain RuleLesson 15: The Chain Rule
Lesson 15: The Chain Rule
 
Hw5sols
Hw5solsHw5sols
Hw5sols
 
Calculus First Test 2011/10/20
Calculus First Test 2011/10/20Calculus First Test 2011/10/20
Calculus First Test 2011/10/20
 
Stepenovanje
StepenovanjeStepenovanje
Stepenovanje
 
Gaussseidelsor
GaussseidelsorGaussseidelsor
Gaussseidelsor
 
Chapter 09
Chapter 09Chapter 09
Chapter 09
 
Pt 2 turunan fungsi eksponen, logaritma, implisit dan cyclometri-d4
Pt 2 turunan fungsi eksponen, logaritma, implisit dan cyclometri-d4Pt 2 turunan fungsi eksponen, logaritma, implisit dan cyclometri-d4
Pt 2 turunan fungsi eksponen, logaritma, implisit dan cyclometri-d4
 
Tugas 1 kalkulus
Tugas 1 kalkulusTugas 1 kalkulus
Tugas 1 kalkulus
 
数式処理ソフトMathematicaで数学の問題を解く
数式処理ソフトMathematicaで数学の問題を解く数式処理ソフトMathematicaで数学の問題を解く
数式処理ソフトMathematicaで数学の問題を解く
 
Derivative of an exponential function
Derivative of an exponential functionDerivative of an exponential function
Derivative of an exponential function
 

En vedette

Description open erp_v_7
Description open erp_v_7Description open erp_v_7
Description open erp_v_7Ab Rafaoui
 
Le tramway à Amiens, florilège de l'absurde
Le tramway à Amiens, florilège de l'absurdeLe tramway à Amiens, florilège de l'absurde
Le tramway à Amiens, florilège de l'absurdeamiens2014
 
Ca 2010 metro cam v2
Ca 2010 metro cam v2Ca 2010 metro cam v2
Ca 2010 metro cam v2amiens2014
 
Le projet municipal 2008
Le projet municipal 2008Le projet municipal 2008
Le projet municipal 2008amiens2014
 
Purdue exhibition
Purdue exhibitionPurdue exhibition
Purdue exhibitionamyc216
 
2 gazelles en 4 l
2 gazelles en 4 l2 gazelles en 4 l
2 gazelles en 4 lacol45
 
Compte rendu : Une rave-party dégénère
Compte rendu : Une rave-party dégénèreCompte rendu : Une rave-party dégénère
Compte rendu : Une rave-party dégénèreLucie Bonnard
 
Eisti - École d'ingénieurs pour bac S 2015-2016
Eisti - École d'ingénieurs pour bac S 2015-2016Eisti - École d'ingénieurs pour bac S 2015-2016
Eisti - École d'ingénieurs pour bac S 2015-2016EISTI
 

En vedette (9)

Description open erp_v_7
Description open erp_v_7Description open erp_v_7
Description open erp_v_7
 
Le tramway à Amiens, florilège de l'absurde
Le tramway à Amiens, florilège de l'absurdeLe tramway à Amiens, florilège de l'absurde
Le tramway à Amiens, florilège de l'absurde
 
Ca 2010 metro cam v2
Ca 2010 metro cam v2Ca 2010 metro cam v2
Ca 2010 metro cam v2
 
Le projet municipal 2008
Le projet municipal 2008Le projet municipal 2008
Le projet municipal 2008
 
Purdue exhibition
Purdue exhibitionPurdue exhibition
Purdue exhibition
 
2 gazelles en 4 l
2 gazelles en 4 l2 gazelles en 4 l
2 gazelles en 4 l
 
Compte rendu : Une rave-party dégénère
Compte rendu : Une rave-party dégénèreCompte rendu : Une rave-party dégénère
Compte rendu : Une rave-party dégénère
 
Eisti - École d'ingénieurs pour bac S 2015-2016
Eisti - École d'ingénieurs pour bac S 2015-2016Eisti - École d'ingénieurs pour bac S 2015-2016
Eisti - École d'ingénieurs pour bac S 2015-2016
 
OpenERP: Comptabilité
OpenERP: ComptabilitéOpenERP: Comptabilité
OpenERP: Comptabilité
 

Similaire à Chapter 08

Emat 213 midterm 1 winter 2006
Emat 213 midterm 1 winter 2006Emat 213 midterm 1 winter 2006
Emat 213 midterm 1 winter 2006akabaka12
 
Engr 213 midterm 1a sol 2010
Engr 213 midterm 1a sol 2010Engr 213 midterm 1a sol 2010
Engr 213 midterm 1a sol 2010akabaka12
 
Engr 213 final sol 2009
Engr 213 final sol 2009Engr 213 final sol 2009
Engr 213 final sol 2009akabaka12
 
Calculus 08 techniques_of_integration
Calculus 08 techniques_of_integrationCalculus 08 techniques_of_integration
Calculus 08 techniques_of_integrationtutulk
 
Lesson 8: Derivatives of Logarithmic and Exponential Functions (worksheet sol...
Lesson 8: Derivatives of Logarithmic and Exponential Functions (worksheet sol...Lesson 8: Derivatives of Logarithmic and Exponential Functions (worksheet sol...
Lesson 8: Derivatives of Logarithmic and Exponential Functions (worksheet sol...Matthew Leingang
 
Lesson 28: Integration by Substitution (worksheet solutions)
Lesson 28: Integration by Substitution (worksheet solutions)Lesson 28: Integration by Substitution (worksheet solutions)
Lesson 28: Integration by Substitution (worksheet solutions)Matthew Leingang
 
Numerical solution of spatiotemporal models from ecology
Numerical solution of spatiotemporal models from ecologyNumerical solution of spatiotemporal models from ecology
Numerical solution of spatiotemporal models from ecologyKyrre Wahl Kongsgård
 
Engr 213 midterm 1b sol 2010
Engr 213 midterm 1b sol 2010Engr 213 midterm 1b sol 2010
Engr 213 midterm 1b sol 2010akabaka12
 
Engr 213 sample midterm 2b sol 2010
Engr 213 sample midterm 2b sol 2010Engr 213 sample midterm 2b sol 2010
Engr 213 sample midterm 2b sol 2010akabaka12
 
Solution Manual : Chapter - 05 Integration
Solution Manual : Chapter - 05 IntegrationSolution Manual : Chapter - 05 Integration
Solution Manual : Chapter - 05 IntegrationHareem Aslam
 
Week 3 [compatibility mode]
Week 3 [compatibility mode]Week 3 [compatibility mode]
Week 3 [compatibility mode]Hazrul156
 

Similaire à Chapter 08 (20)

Emat 213 midterm 1 winter 2006
Emat 213 midterm 1 winter 2006Emat 213 midterm 1 winter 2006
Emat 213 midterm 1 winter 2006
 
Leidy rivadeneira deber_1
Leidy rivadeneira deber_1Leidy rivadeneira deber_1
Leidy rivadeneira deber_1
 
Engr 213 midterm 1a sol 2010
Engr 213 midterm 1a sol 2010Engr 213 midterm 1a sol 2010
Engr 213 midterm 1a sol 2010
 
Engr 213 final sol 2009
Engr 213 final sol 2009Engr 213 final sol 2009
Engr 213 final sol 2009
 
Ism et chapter_6
Ism et chapter_6Ism et chapter_6
Ism et chapter_6
 
Sect1 2
Sect1 2Sect1 2
Sect1 2
 
Week 2
Week 2 Week 2
Week 2
 
125 5.2
125 5.2125 5.2
125 5.2
 
Calculus 08 techniques_of_integration
Calculus 08 techniques_of_integrationCalculus 08 techniques_of_integration
Calculus 08 techniques_of_integration
 
Lesson 8: Derivatives of Logarithmic and Exponential Functions (worksheet sol...
Lesson 8: Derivatives of Logarithmic and Exponential Functions (worksheet sol...Lesson 8: Derivatives of Logarithmic and Exponential Functions (worksheet sol...
Lesson 8: Derivatives of Logarithmic and Exponential Functions (worksheet sol...
 
Lesson 28: Integration by Substitution (worksheet solutions)
Lesson 28: Integration by Substitution (worksheet solutions)Lesson 28: Integration by Substitution (worksheet solutions)
Lesson 28: Integration by Substitution (worksheet solutions)
 
Tugas akhir matematika kelompok 1
Tugas akhir matematika kelompok 1Tugas akhir matematika kelompok 1
Tugas akhir matematika kelompok 1
 
Numerical solution of spatiotemporal models from ecology
Numerical solution of spatiotemporal models from ecologyNumerical solution of spatiotemporal models from ecology
Numerical solution of spatiotemporal models from ecology
 
Engr 213 midterm 1b sol 2010
Engr 213 midterm 1b sol 2010Engr 213 midterm 1b sol 2010
Engr 213 midterm 1b sol 2010
 
Engr 213 sample midterm 2b sol 2010
Engr 213 sample midterm 2b sol 2010Engr 213 sample midterm 2b sol 2010
Engr 213 sample midterm 2b sol 2010
 
Solution Manual : Chapter - 05 Integration
Solution Manual : Chapter - 05 IntegrationSolution Manual : Chapter - 05 Integration
Solution Manual : Chapter - 05 Integration
 
11365.integral 2
11365.integral 211365.integral 2
11365.integral 2
 
Week 3 [compatibility mode]
Week 3 [compatibility mode]Week 3 [compatibility mode]
Week 3 [compatibility mode]
 
Chapter 04
Chapter 04Chapter 04
Chapter 04
 
Lista de derivadas e integrais
Lista de derivadas e integraisLista de derivadas e integrais
Lista de derivadas e integrais
 

Plus de ramiz100111

Plus de ramiz100111 (8)

Chapter 14
Chapter 14Chapter 14
Chapter 14
 
Chapter 16
Chapter 16Chapter 16
Chapter 16
 
Chapter 10
Chapter 10Chapter 10
Chapter 10
 
Chapter 12
Chapter 12Chapter 12
Chapter 12
 
Chapter 07
Chapter 07Chapter 07
Chapter 07
 
Chapter 06
Chapter 06Chapter 06
Chapter 06
 
Chapter 01
Chapter 01Chapter 01
Chapter 01
 
Appendix a page_524
Appendix a page_524Appendix a page_524
Appendix a page_524
 

Dernier

social pharmacy d-pharm 1st year by Pragati K. Mahajan
social pharmacy d-pharm 1st year by Pragati K. Mahajansocial pharmacy d-pharm 1st year by Pragati K. Mahajan
social pharmacy d-pharm 1st year by Pragati K. Mahajanpragatimahajan3
 
1029 - Danh muc Sach Giao Khoa 10 . pdf
1029 -  Danh muc Sach Giao Khoa 10 . pdf1029 -  Danh muc Sach Giao Khoa 10 . pdf
1029 - Danh muc Sach Giao Khoa 10 . pdfQucHHunhnh
 
Kisan Call Centre - To harness potential of ICT in Agriculture by answer farm...
Kisan Call Centre - To harness potential of ICT in Agriculture by answer farm...Kisan Call Centre - To harness potential of ICT in Agriculture by answer farm...
Kisan Call Centre - To harness potential of ICT in Agriculture by answer farm...Krashi Coaching
 
APM Welcome, APM North West Network Conference, Synergies Across Sectors
APM Welcome, APM North West Network Conference, Synergies Across SectorsAPM Welcome, APM North West Network Conference, Synergies Across Sectors
APM Welcome, APM North West Network Conference, Synergies Across SectorsAssociation for Project Management
 
The Most Excellent Way | 1 Corinthians 13
The Most Excellent Way | 1 Corinthians 13The Most Excellent Way | 1 Corinthians 13
The Most Excellent Way | 1 Corinthians 13Steve Thomason
 
IGNOU MSCCFT and PGDCFT Exam Question Pattern: MCFT003 Counselling and Family...
IGNOU MSCCFT and PGDCFT Exam Question Pattern: MCFT003 Counselling and Family...IGNOU MSCCFT and PGDCFT Exam Question Pattern: MCFT003 Counselling and Family...
IGNOU MSCCFT and PGDCFT Exam Question Pattern: MCFT003 Counselling and Family...PsychoTech Services
 
Software Engineering Methodologies (overview)
Software Engineering Methodologies (overview)Software Engineering Methodologies (overview)
Software Engineering Methodologies (overview)eniolaolutunde
 
BAG TECHNIQUE Bag technique-a tool making use of public health bag through wh...
BAG TECHNIQUE Bag technique-a tool making use of public health bag through wh...BAG TECHNIQUE Bag technique-a tool making use of public health bag through wh...
BAG TECHNIQUE Bag technique-a tool making use of public health bag through wh...Sapna Thakur
 
Arihant handbook biology for class 11 .pdf
Arihant handbook biology for class 11 .pdfArihant handbook biology for class 11 .pdf
Arihant handbook biology for class 11 .pdfchloefrazer622
 
Measures of Central Tendency: Mean, Median and Mode
Measures of Central Tendency: Mean, Median and ModeMeasures of Central Tendency: Mean, Median and Mode
Measures of Central Tendency: Mean, Median and ModeThiyagu K
 
Q4-W6-Restating Informational Text Grade 3
Q4-W6-Restating Informational Text Grade 3Q4-W6-Restating Informational Text Grade 3
Q4-W6-Restating Informational Text Grade 3JemimahLaneBuaron
 
Student login on Anyboli platform.helpin
Student login on Anyboli platform.helpinStudent login on Anyboli platform.helpin
Student login on Anyboli platform.helpinRaunakKeshri1
 
Grant Readiness 101 TechSoup and Remy Consulting
Grant Readiness 101 TechSoup and Remy ConsultingGrant Readiness 101 TechSoup and Remy Consulting
Grant Readiness 101 TechSoup and Remy ConsultingTechSoup
 
General AI for Medical Educators April 2024
General AI for Medical Educators April 2024General AI for Medical Educators April 2024
General AI for Medical Educators April 2024Janet Corral
 
1029-Danh muc Sach Giao Khoa khoi 6.pdf
1029-Danh muc Sach Giao Khoa khoi  6.pdf1029-Danh muc Sach Giao Khoa khoi  6.pdf
1029-Danh muc Sach Giao Khoa khoi 6.pdfQucHHunhnh
 
Z Score,T Score, Percential Rank and Box Plot Graph
Z Score,T Score, Percential Rank and Box Plot GraphZ Score,T Score, Percential Rank and Box Plot Graph
Z Score,T Score, Percential Rank and Box Plot GraphThiyagu K
 
SOCIAL AND HISTORICAL CONTEXT - LFTVD.pptx
SOCIAL AND HISTORICAL CONTEXT - LFTVD.pptxSOCIAL AND HISTORICAL CONTEXT - LFTVD.pptx
SOCIAL AND HISTORICAL CONTEXT - LFTVD.pptxiammrhaywood
 
Disha NEET Physics Guide for classes 11 and 12.pdf
Disha NEET Physics Guide for classes 11 and 12.pdfDisha NEET Physics Guide for classes 11 and 12.pdf
Disha NEET Physics Guide for classes 11 and 12.pdfchloefrazer622
 
microwave assisted reaction. General introduction
microwave assisted reaction. General introductionmicrowave assisted reaction. General introduction
microwave assisted reaction. General introductionMaksud Ahmed
 

Dernier (20)

social pharmacy d-pharm 1st year by Pragati K. Mahajan
social pharmacy d-pharm 1st year by Pragati K. Mahajansocial pharmacy d-pharm 1st year by Pragati K. Mahajan
social pharmacy d-pharm 1st year by Pragati K. Mahajan
 
1029 - Danh muc Sach Giao Khoa 10 . pdf
1029 -  Danh muc Sach Giao Khoa 10 . pdf1029 -  Danh muc Sach Giao Khoa 10 . pdf
1029 - Danh muc Sach Giao Khoa 10 . pdf
 
Kisan Call Centre - To harness potential of ICT in Agriculture by answer farm...
Kisan Call Centre - To harness potential of ICT in Agriculture by answer farm...Kisan Call Centre - To harness potential of ICT in Agriculture by answer farm...
Kisan Call Centre - To harness potential of ICT in Agriculture by answer farm...
 
APM Welcome, APM North West Network Conference, Synergies Across Sectors
APM Welcome, APM North West Network Conference, Synergies Across SectorsAPM Welcome, APM North West Network Conference, Synergies Across Sectors
APM Welcome, APM North West Network Conference, Synergies Across Sectors
 
The Most Excellent Way | 1 Corinthians 13
The Most Excellent Way | 1 Corinthians 13The Most Excellent Way | 1 Corinthians 13
The Most Excellent Way | 1 Corinthians 13
 
IGNOU MSCCFT and PGDCFT Exam Question Pattern: MCFT003 Counselling and Family...
IGNOU MSCCFT and PGDCFT Exam Question Pattern: MCFT003 Counselling and Family...IGNOU MSCCFT and PGDCFT Exam Question Pattern: MCFT003 Counselling and Family...
IGNOU MSCCFT and PGDCFT Exam Question Pattern: MCFT003 Counselling and Family...
 
Software Engineering Methodologies (overview)
Software Engineering Methodologies (overview)Software Engineering Methodologies (overview)
Software Engineering Methodologies (overview)
 
BAG TECHNIQUE Bag technique-a tool making use of public health bag through wh...
BAG TECHNIQUE Bag technique-a tool making use of public health bag through wh...BAG TECHNIQUE Bag technique-a tool making use of public health bag through wh...
BAG TECHNIQUE Bag technique-a tool making use of public health bag through wh...
 
Arihant handbook biology for class 11 .pdf
Arihant handbook biology for class 11 .pdfArihant handbook biology for class 11 .pdf
Arihant handbook biology for class 11 .pdf
 
Measures of Central Tendency: Mean, Median and Mode
Measures of Central Tendency: Mean, Median and ModeMeasures of Central Tendency: Mean, Median and Mode
Measures of Central Tendency: Mean, Median and Mode
 
Mattingly "AI & Prompt Design: Structured Data, Assistants, & RAG"
Mattingly "AI & Prompt Design: Structured Data, Assistants, & RAG"Mattingly "AI & Prompt Design: Structured Data, Assistants, & RAG"
Mattingly "AI & Prompt Design: Structured Data, Assistants, & RAG"
 
Q4-W6-Restating Informational Text Grade 3
Q4-W6-Restating Informational Text Grade 3Q4-W6-Restating Informational Text Grade 3
Q4-W6-Restating Informational Text Grade 3
 
Student login on Anyboli platform.helpin
Student login on Anyboli platform.helpinStudent login on Anyboli platform.helpin
Student login on Anyboli platform.helpin
 
Grant Readiness 101 TechSoup and Remy Consulting
Grant Readiness 101 TechSoup and Remy ConsultingGrant Readiness 101 TechSoup and Remy Consulting
Grant Readiness 101 TechSoup and Remy Consulting
 
General AI for Medical Educators April 2024
General AI for Medical Educators April 2024General AI for Medical Educators April 2024
General AI for Medical Educators April 2024
 
1029-Danh muc Sach Giao Khoa khoi 6.pdf
1029-Danh muc Sach Giao Khoa khoi  6.pdf1029-Danh muc Sach Giao Khoa khoi  6.pdf
1029-Danh muc Sach Giao Khoa khoi 6.pdf
 
Z Score,T Score, Percential Rank and Box Plot Graph
Z Score,T Score, Percential Rank and Box Plot GraphZ Score,T Score, Percential Rank and Box Plot Graph
Z Score,T Score, Percential Rank and Box Plot Graph
 
SOCIAL AND HISTORICAL CONTEXT - LFTVD.pptx
SOCIAL AND HISTORICAL CONTEXT - LFTVD.pptxSOCIAL AND HISTORICAL CONTEXT - LFTVD.pptx
SOCIAL AND HISTORICAL CONTEXT - LFTVD.pptx
 
Disha NEET Physics Guide for classes 11 and 12.pdf
Disha NEET Physics Guide for classes 11 and 12.pdfDisha NEET Physics Guide for classes 11 and 12.pdf
Disha NEET Physics Guide for classes 11 and 12.pdf
 
microwave assisted reaction. General introduction
microwave assisted reaction. General introductionmicrowave assisted reaction. General introduction
microwave assisted reaction. General introduction
 

Chapter 08

  • 1. January 27, 2005 11:45 L24-CH08 Sheet number 1 Page number 337 black CHAPTER 8 Principles of Integral Valuation EXERCISE SET 8.1 1 1 1 1. u = 4 − 2x, du = −2dx, − u3 du = − u4 + C = − (4 − 2x)4 + C 2 8 8 3 √ 2. u = 4 + 2x, du = 2dx, u du = u3/2 + C = (4 + 2x)3/2 + C 2 1 1 1 3. u = x2 , du = 2xdx, sec2 u du = tan u + C = tan(x2 ) + C 2 2 2 4. u = x2 , du = 2xdx, 2 tan u du = −2 ln | cos u | + C = −2 ln | cos(x2 )| + C 1 du 1 1 5. u = 2 + cos 3x, du = −3 sin 3xdx, − = − ln |u| + C = − ln(2 + cos 3x) + C 3 u 3 3 2 2 1 du 1 1 2 6. u = x, du = dx, = tan−1 u + C = tan−1 x + C 3 3 6 1 + u2 6 6 3 7. u = ex , du = ex dx, sinh u du = cosh u + C = cosh ex + C 1 8. u = ln x, du = dx, sec u tan u du = sec u + C = sec(ln x) + C x 9. u = tan x, du = sec2 xdx, eu du = eu + C = etan x + C 1 du 1 1 10. u = x2 , du = 2xdx, √ = sin−1 u + C = sin−1 (x2 ) + C 2 1−u 2 2 2 1 1 6 1 11. u = cos 5x, du = −5 sin 5xdx, − u5 du = − u + C = − cos6 5x + C 5 30 30 √ du 1 + 1 + u2 1 + 1 + sin2 x 12. u = sin x, du = cos x dx, √ = − ln + C = − ln +C u u2 + 1 u sin x du 13. u = ex , du = ex dx, √ = ln u + u2 + 4 + C = ln ex + e2x + 4 + C 4 + u2 1 −1 14. u = tan−1 x, du = dx, eu du = eu + C = etan x +C 1 + x2 √ 1 √ 15. u = x − 1, du = √ dx, 2 eu du = 2eu + C = 2e x−1 +C 2 x−1 1 1 1 16. u = x2 + 2x, du = (2x + 2)dx, cot u du = ln | sin u| + C = ln sin |x2 + 2x| + C 2 2 2 √ 1 √ 17. u = x, du = √ dx, 2 cosh u du = 2 sinh u + C = 2 sinh x + C 2 x 337
  • 2. January 27, 2005 11:45 L24-CH08 Sheet number 2 Page number 338 black 338 Chapter 8 dx du 1 1 18. u = ln x, du = , 2 =− +C =− +C x u u ln x √ 1 2 du 2 −u ln 3 2 −√x 19. u = x, du = √ dx, =2 e−u ln 3 du = − e +C =− 3 +C 2 x 3u ln 3 ln 3 20. u = sin θ, du = cos θdθ, sec u tan u du = sec u + C = sec(sin θ) + C 2 2 1 1 1 2 21. u = , du = − 2 dx, − csch2 u du = coth u + C = coth + C x x 2 2 2 x dx 22. √ = ln x + x2 − 4 + C x2 − 4 du 1 2+u 1 2 + e−x 23. u = e−x , du = −e−x dx, − = − ln + C = − ln +C 4−u 2 4 2−u 4 2 − e−x 1 24. u = ln x, du = dx, cos u du = sin u + C = sin(ln x) + C x ex dx du 25. u = ex , du = ex dx, √ = √ = sin−1 u + C = sin−1 ex + C 1 − e2x 1 − u2 1 26. u = x−1/2 , du = − dx, − 2 sinh u du = −2 cosh u + C = −2 cosh(x−1/2 ) + C 2x3/2 1 du 1 1 1 27. u = x2 , du = 2xdx, = sin u du = − cos u + C = − cos(x2 ) + C 2 csc u 2 2 2 2du 28. 2u = ex , 2du = ex dx, √ = sin−1 u + C = sin−1 (ex /2) + C 4 − 4u2 29. 4−x = e−x 2 2 ln 4 , u = −x2 ln 4, du = −2x ln 4 dx = −x ln 16 dx, 1 1 u 1 −x2 ln 4 1 −x2 − eu du = − e +C =− e +C =− 4 +C ln 16 ln 16 ln 16 ln 16 1 πx ln 2 1 30. 2πx = eπx ln 2 , 2πx dx = e +C = 2πx + C π ln 2 π ln 2 1 2 1 31. (a) u = sin x, du = cos x dx, u du = u + C = sin2 x + C 2 2 1 1 1 (b) sin x cos x dx = sin 2x dx = − cos 2x + C = − (cos2 x − sin2 x) + C 2 4 4 1 1 1 1 (c) − (cos2 x − sin2 x) + C = − (1 − sin2 x − sin2 x) + C = − + sin2 x + C, 4 4 4 2 and this is the same as the answer in part (a) except for the constants. 1 1 32. (a) sech 2x = = (now multiply top and bottom by sech2 x) cosh 2x cosh2 x + sinh2 x sech2 x = 1 + tanh2 x
  • 3. January 27, 2005 11:45 L24-CH08 Sheet number 3 Page number 339 black Exercise Set 8.2 339 sech2 x (b) sech2x dx = dx = tan−1 (tanh x) + C, or, replacing 2x with x, 1 + tanh2 x sechx dx = tan−1 (tanh(x/2)) + C 1 2 2ex (c) sech x = = x = 2x cosh x e + e−x e +1 ex (d) sech x dx = 2 dx = 2 tan−1 (ex ) + C e2x+1 sec2 x 1 1 33. (a) = = tan x cos2 x tan x cos x sin x 1 1 1 sec2 x 1 (b) csc 2x = = = , so csc 2x dx = ln tan x + C sin 2x 2 sin x cos x 2 tan x 2 1 1 (c) sec x = = = csc(π/2 − x), so cos x sin(π/2 − x) 1 sec x dx = csc(π/2 − x) dx = − ln tan(π/2 − x) + C 2 EXERCISE SET 8.2 1. u = x, dv = e−2x dx, du = dx, v = − 1 e−2x ; 2 1 1 −2x 1 1 xe−2x dx = − xe−2x + e dx = − xe−2x − e−2x + C 2 2 2 4 1 3x 1 3x 1 1 3x 1 3x 2. u = x, dv = e3x dx, du = dx, v = e ; xe3x dx = xe − e3x dx = xe − e + C 3 3 3 3 9 3. u = x2 , dv = ex dx, du = 2x dx, v = ex ; x2 ex dx = x2 ex − 2 xex dx. For xex dx use u = x, dv = ex dx, du = dx, v = ex to get xex dx = xex − ex + C1 so x2 ex dx = x2 ex − 2xex + 2ex + C 1 1 4. u = x2 , dv = e−2x dx, du = 2x dx, v = − e−2x ; x2 e−2x dx = − x2 e−2x + xe−2x dx 2 2 For xe−2x dx use u = x, dv = e−2x dx to get 1 1 1 1 xe−2x dx = − xe−2x + e−2x dx = − xe−2x − e−2x + C 2 2 2 4 1 1 1 so x2 e−2x dx = − x2 e−2x − xe−2x − e−2x + C 2 2 4 1 5. u = x, dv = sin 3x dx, du = dx, v = − cos 3x; 3 1 1 1 1 x sin 3x dx = − x cos 3x + cos 3x dx = − x cos 3x + sin 3x + C 3 3 3 9
  • 4. January 27, 2005 11:45 L24-CH08 Sheet number 4 Page number 340 black 340 Chapter 8 1 6. u = x, dv = cos 2x dx, du = dx, v = sin 2x; 2 1 1 1 1 x cos 2x dx = x sin 2x − sin 2x dx = x sin 2x + cos 2x + C 2 2 2 4 7. u = x2 , dv = cos x dx, du = 2x dx, v = sin x; x2 cos x dx = x2 sin x − 2 x sin x dx For x sin x dx use u = x, dv = sin x dx to get x sin x dx = −x cos x + sin x + C1 so x2 cos x dx = x2 sin x + 2x cos x − 2 sin x + C 8. u = x2 , dv = sin x dx, du = 2x dx, v = − cos x; x2 sin x dx = −x2 cos x + 2 x cos x dx; for x cos x dx use u = x, dv = cos x dx to get x cos x dx = x sin x + cos x + C1 so x2 sin x dx = −x2 cos x + 2x sin x + 2 cos x + C 1 1 1 2 1 1 2 1 9. u = ln x, dv = x dx, du = dx, v = x2 ; x ln x dx = x ln x − x dx = x ln x − x2 + C x 2 2 2 2 4 √ 1 2 10. u = ln x, dv = dx, v = x3/2 ; x dx, du = x 3 √ 2 3/2 2 2 4 x ln x dx = x ln x − x1/2 dx = x3/2 ln x − x3/2 + C 3 3 3 9 ln x 11. u = (ln x)2 , dv = dx, du = 2 dx, v = x; (ln x)2 dx = x(ln x)2 − 2 ln x dx. x Use u = ln x, dv = dx to get ln x dx = x ln x − dx = x ln x − x + C1 so (ln x)2 dx = x(ln x)2 − 2x ln x + 2x + C 1 1 √ ln x √ 1 √ √ 12. u = ln x, dv = √ dx, du = dx, v = 2 x; √ dx = 2 x ln x−2 √ dx = 2 x ln x−4 x+C x x x x 3 3x 13. u = ln(3x − 2), dv = dx, du = dx, v = x; ln(3x − 2)dx = x ln(3x − 2) − dx 3x − 2 3x − 2 3x 2 2 but dx = 1+ dx = x + ln(3x − 2) + C1 so 3x − 2 3x − 2 3 2 ln(3x − 2)dx = x ln(3x − 2) − x − ln(3x − 2) + C 3 2x x2 14. u = ln(x2 + 4), dv = dx, du = 2+4 dx, v = x; ln(x2 + 4)dx = x ln(x2 + 4) − 2 dx x x2 + 4 x2 4 x but dx = 1− dx = x − 2 tan−1 + C1 so x2 +4 x2 + 4 2 x ln(x2 + 4)dx = x ln(x2 + 4) − 2x + 4 tan−1 +C 2
  • 5. January 27, 2005 11:45 L24-CH08 Sheet number 5 Page number 341 black Exercise Set 8.2 341 √ 15. u = sin−1 x, dv = dx, du = 1/ 1 − x2 dx, v = x; sin−1 x dx = x sin−1 x − x/ 1 − x2 dx = x sin−1 x + 1 − x2 + C 2 16. u = cos−1 (2x), dv = dx, du = − √ dx, v = x; 1 − 4x2 2x 1 cos−1 (2x)dx = x cos−1 (2x) + √ dx = x cos−1 (2x) − 1 − 4x2 + C 1 − 4x2 2 3 17. u = tan−1 (3x), dv = dx, du = dx, v = x; 1 + 9x2 3x 1 tan−1 (3x)dx = x tan−1 (3x) − dx = x tan−1 (3x) − ln(1 + 9x2 ) + C 1 + 9x2 6 1 1 1 1 x2 18. u = tan−1 x, dv = x dx, du = dx, v = x2 ; x tan−1 x dx = x2 tan−1 x − dx 1 + x2 2 2 2 1 + x2 x2 1 but dx = 1− dx = x − tan−1 x + C1 so 1 + x2 1 + x2 1 1 1 x tan−1 x dx = x2 tan−1 x − x + tan−1 x + C 2 2 2 19. u = ex , dv = sin x dx, du = ex dx, v = − cos x; ex sin x dx = −ex cos x + ex cos x dx. For ex cos x dx use u = ex , dv = cos x dx to get ex cos x = ex sin x − ex sin x dx so ex sin x dx = −ex cos x + ex sin x − ex sin x dx, 1 x 2 ex sin x dx = ex (sin x − cos x) + C1 , ex sin x dx = e (sin x − cos x) + C 2 1 20. u = e3x , dv = cos 2x dx, du = 3e3x dx, v = sin 2x; 2 1 3x 3 e3x cos 2x dx = e sin 2x − e3x sin 2x dx. Use u = e3x , dv = sin 2x dx to get 2 2 1 3 e3x sin 2x dx = − e3x cos 2x + e3x cos 2x dx, so 2 2 1 3 9 e3x cos 2x dx = e3x sin 2x + e3x cos 2x − e3x cos 2x dx, 2 4 4 13 1 1 3x e3x cos 2x dx = e3x (2 sin 2x + 3 cos 2x) + C1 , e3x cos 2x dx = e (2 sin 2x + 3 cos 2x) + C 4 4 13 1 21. u = eax , dv = sin bx dx, du = aeax dx, v = − cos bx (b = 0); b 1 ax a eax sin bx dx = − e cos bx + eax cos bx dx. Use u = eax , dv = cos bx dx to get b b 1 a eax cos bx dx = eax sin bx − eax sin bx dx so b b 1 a a2 eax sin bx dx = − eax cos bx + 2 eax sin bx − 2 eax sin bx dx, b b b eax eax sin bx dx = 2 (a sin bx − b cos bx) + C a + b2
  • 6. January 27, 2005 11:45 L24-CH08 Sheet number 6 Page number 342 black 342 Chapter 8 e−3θ 22. From Exercise 21 with a = −3, b = 5, x = θ, answer = √ (−3 sin 5θ − 5 cos 5θ) + C 34 cos(ln x) 23. u = sin(ln x), dv = dx, du = dx, v = x; x sin(ln x)dx = x sin(ln x) − cos(ln x)dx. Use u = cos(ln x), dv = dx to get cos(ln x)dx = x cos(ln x) + sin(ln x)dx so sin(ln x)dx = x sin(ln x) − x cos(ln x) − sin(ln x)dx, 1 sin(ln x)dx = x[sin(ln x) − cos(ln x)] + C 2 1 24. u = cos(ln x), dv = dx, du = − sin(ln x)dx, v = x; x cos(ln x)dx = x cos(ln x) + sin(ln x)dx. Use u = sin(ln x), dv = dx to get sin(ln x)dx = x sin(ln x) − cos(ln x)dx so cos(ln x)dx = x cos(ln x) + x sin(ln x) − cos(ln x)dx, 1 cos(ln x)dx = x[cos(ln x) + sin(ln x)] + C 2 25. u = x, dv = sec2 x dx, du = dx, v = tan x; sin x x sec2 x dx = x tan x − tan x dx = x tan x − dx = x tan x + ln | cos x| + C cos x 26. u = x, dv = tan2 x dx = (sec2 x − 1)dx, du = dx, v = tan x − x; x tan2 x dx = x tan x − x2 − (tan x − x)dx 1 1 = x tan x − x2 + ln | cos x| + x2 + C = x tan x − x2 + ln | cos x| + C 2 2 2 1 x2 27. u = x2 , dv = xex dx, du = 2x dx, v = e ; 2 2 1 2 x2 2 1 2 x2 1 x2 x3 ex dx = x e − xex dx = x e − e +C 2 2 2 1 1 28. u = xex , dv = dx, du = (x + 1)ex dx, v = − ; (x + 1)2 x+1 xex xex xex ex dx = − + ex dx = − + ex + C = +C (x + 1)2 x+1 x+1 x+1 1 2x 29. u = x, dv = e2x dx, du = dx, v = e ; 2 2 2 2 2 1 2x 1 1 1 xe2x dx = xe − e2x dx = e4 − e2x = e4 − (e4 − 1) = (3e4 + 1)/4 0 2 0 2 0 4 0 4
  • 7. January 27, 2005 11:45 L24-CH08 Sheet number 7 Page number 343 black Exercise Set 8.2 343 1 30. u = x, dv = e−5x dx, du = dx, v = − e−5x ; 5 1 1 1 1 1 xe−5x dx = − xe−5x + e−5x dx 0 5 0 5 0 1 1 1 1 1 = − e−5 − e−5x = − e−5 − (e−5 − 1) = (1 − 6e−5 )/25 5 25 0 5 25 1 1 31. u = ln x, dv = x2 dx, du = dx, v = x3 ; x 3 e e e e 1 3 1 1 3 1 3 1 3 1 3 x2 ln x dx = x ln x − x2 dx = e − x = e − (e − 1) = (2e3 + 1)/9 1 3 1 3 1 3 9 1 3 9 1 1 1 32. u = ln x, dv = 2 dx, du = dx, v = − ; x x x e e e ln x 1 1 √ 2 dx = − ln x √ + √ dx e x x e e x2 √ 1 1 √ 1 e 1 1 1 1 3 e−4 = − + √ ln e − √ =− + √ − +√ = e e x e e 2 e e e 2e 1 33. u = ln(x + 2), dv = dx, du = dx, v = x; x+2 1 1 1 1 x 2 ln(x + 2)dx = x ln(x + 2) − dx = ln 3 + ln 1 − 1− dx −1 −1 −1 x+2 −1 x+2 1 = ln 3 − [x − 2 ln(x + 2)] = ln 3 − (1 − 2 ln 3) + (−1 − 2 ln 1) = 3 ln 3 − 2 −1 1 34. u = sin−1 x, dv = dx, du = √ dx, v = x; 1 − x2 √ 3/2 √ 3/2 √ 3/2 √ √ √ 3/2 −1 −1 x 3 3 sin x dx = x sin x − √ dx = sin−1 + 1 − x2 0 0 0 1 − x2 2 2 0 √ √ 3 π 1 π 3 1 = + −1= − 2 3 2 6 2 √ 1 35. u = sec−1 θ, dv = dθ, du = √ dθ, v = θ; 2θ θ − 1 4 √ √1 4 1 √ 4 √ 4 sec−1 θdθ = θ sec−1 dθ = 4 sec−1 2 − 2 sec−1 2 − θ − 1 θ − √ 2 2 2 θ−1 2 2 π π √ 5π √ =4 −2 − 3+1= − 3+1 3 4 6 1 1 36. u = sec−1 x, dv = x dx, du = √ dx, v = x2 ; x x2−1 2 2 2 2 1 2 1 x x sec−1 x dx = x sec−1 x − √ dx 1 2 1 2 1 x2 − 1 1 1 2 √ = [(4)(π/3) − (1)(0)] − x2 − 1 = 2π/3 − 3/2 2 2 1
  • 8. January 27, 2005 11:45 L24-CH08 Sheet number 8 Page number 344 black 344 Chapter 8 1 37. u = x, dv = sin 2x dx, du = dx, v = − cos 2x; 2 π π π π 1 1 1 x sin 2x dx = − x cos 2x + cos 2x dx = −π/2 + sin 2x = −π/2 0 2 0 2 0 4 0 π π π π 1 2 π2 38. (x + x cos x)dx = x + x cos x dx = + x cos x dx; 0 2 0 0 2 0 u = x, dv = cos x dx, du = dx, v = sin x π π π π π x cos x dx = x sin x − sin x dx = cos x = −2 so (x + x cos x)dx = π 2 /2 − 2 0 0 0 0 0 √ √ 1 2 39. u = tan−1 x, dv = xdx, du = √ dx, v = x3/2 ; 2 x(1 + x) 3 3 √ √ 2 3/2 √ 3 1 3 x x tan−1 xdx = x tan−1 x − dx 1 3 1 3 1 1+x 2 3/2 √ 3 1 3 1 = x tan−1 x − 1− dx 3 1 3 1 1+x 2 3/2 √ 1 1 3 √ = x tan−1 x − x + ln |1 + x| = (2 3π − π/2 − 2 + ln 2)/3 3 3 3 1 2x 40. u = ln(x2 + 1), dv = dx, du = dx, v = x; x2 + 1 2 2 2 2 2x2 1 ln(x2 + 1)dx = x ln(x2 + 1) − dx = 2 ln 5 − 2 1− dx 0 0 0 x2+1 0 x2 + 1 2 = 2 ln 5 − 2(x − tan−1 x) = 2 ln 5 − 4 + 2 tan−1 2 0 √ 41. t = x, t2 = x, dx = 2t dt √ x (a) e dx = 2 tet dt; u = t, dv = et dt, du = dt, v = et , √ √ √ e x dx = 2tet − 2 et dt = 2(t − 1)et + C = 2( x − 1)e x + C √ (b) cos x dx = 2 t cos t dt; u = t, dv = cos tdt, du = dt, v = sin t, √ √ √ √ cos x dx = 2t sin t − 2 sin tdt = 2t sin t + 2 cos t + C = 2 x sin x + 2 cos x + C
  • 9. January 27, 2005 11:45 L24-CH08 Sheet number 9 Page number 345 black Exercise Set 8.2 345 42. Let q1 (x), q2 (x), q3 (x) denote successive antiderivatives of q(x), so that q3 (x) = q2 (x), q2 (x) = q1 (x), q1 (x) = q(x). Let p(x) = ax2 + bx + c. Repeated Repeated Differentiation Antidifferentiation ax2 + bx + c q(x) + 2ax + b q1 (x) − 2a q2 (x) + 0 q3 (x) Then p(x)q(x) dx = (ax2 + bx + c)q1 (x) − (2ax + b)q2 (x) + 2aq3 (x) + C. Check: d [(ax2 +bx + c)q1 (x) − (2ax + b)q2 (x) + 2aq3 (x)] dx = (2ax + b)q1 (x) + (ax2 + bx + c)q(x) − 2aq2 (x) − (2ax + b)q1 (x) + 2aq2 (x) = p(x)q(x) 43. Repeated Repeated Differentiation Antidifferentiation 3x2 − x + 2 e−x + 6x − 1 −e−x − 6 e−x + 0 −e−x (3x2 − x + 2)e−x = −(3x2 − x + 2)e−x − (6x − 1)e−x − 6e−x + C = −e−x [3x2 + 5x + 7] + C 44. Repeated Repeated Differentiation Antidifferentiation x2 + x + 1 sin x + 2x + 1 − cos x − 2 − sin x + 0 cos x (x2 + x + 1) sin x dx = −(x2 + x + 1) cos x + (2x + 1) sin x + 2 cos x + C = −(x2 + x − 1) cos x + (2x + 1) sin x + C
  • 10. January 27, 2005 11:45 L24-CH08 Sheet number 10 Page number 346 black 346 Chapter 8 45. Repeated Repeated Differentiation Antidifferentiation 4x4 sin 2x + 1 16x3 − cos 2x 2 − 1 48x 2 − sin 2x + 4 1 96x cos 2x 8 − 1 96 sin 2x 16 + 1 0 − cos 2x 32 4x4 sin 2x dx = (−2x4 + 6x2 − 3) cos 2x + −(4x3 + 6x) sin 2x + C 46. Repeated Repeated Differentiation Antidifferentiation √ x3 2x + 1 + 1 3x 2 (2x + 1)3/2 3 − 1 6x (2x + 1)5/2 15 + 1 6 (2x + 1)7/2 105 − 1 0 (2x + 1)9/2 945 √ 1 1 2 2 x3 2x + 1 dx = x3 (2x + 1)3/2 − x2 (2x + 1)5/2 + x(2x + 1)7/2 − (2x + 1)9/2 + C 3 5 35 315 47. (a) We perform a single integration by parts: u = cos x, dv = sin x dx, du = − sin x dx, v = − cos x, sin x cos x dx = − cos2 x − sin x cos x dx. Thus 1 2 sin x cos x dx = − cos2 x + C, sin x cos x dx = − cos2 x + C 2 1 2 1 (b) u = sin x, du = cos x dx, sin x cos x dx = u du = u + C = sin2 x + C 2 2 x 48. (a) u = x2 , dv = √ , du = 2x dx, v = x2 + 1, x2 +1 1 x3 1 1 √ 2 1 1√ 2 √ dx = x2 x2 + 1 − 2x x2 + 1 dx = 2 − (x2 + 1)3/2 =− 2+ 0 x2 + 1 0 0 3 0 3 3
  • 11. January 27, 2005 11:45 L24-CH08 Sheet number 11 Page number 347 black Exercise Set 8.2 347 √ √ 2 2 x 1 3 (b) u = x2 + 1, du = √ dx, (u − 1) du = 2 u −u x2 + 1 1 3 1 2√ √ 1 1√ 2 = 2− 2− +1=− 2+ . 3 3 3 3 e e 49. (a) A = ln x dx = (x ln x − x) =1 1 1 e e (b) V = π (ln x)2 dx = π (x(ln x)2 − 2x ln x + 2x) = π(e − 2) 1 1 π/2 π/2 π/2 π/2 1 2 π2 50. A = (x − x sin x)dx = x − x sin x dx = − (−x cos x + sin x) = π 2 /8 − 1 0 2 0 0 8 0 π π 51. V = 2π x sin x dx = 2π(−x cos x + sin x) = 2π 2 0 0 π/2 π/2 52. V = 2π x cos x dx = 2π(cos x + x sin x) = π(π − 2) 0 0 π 53. distance = t3 sin tdt; 0 Repeated Repeated Differentiation Antidifferentiation t3 sin t + 3t2 − cos t − 6t − sin t + 6 cos t − 0 sin t π π t3 sin t dx = [(−t3 cos t + 3t2 sin t + 6t cos t − 6 sin t)] = π 3 − 6π 0 0 1 54. u = 2t, dv = sin(kωt)dt, du = 2dt, v = − cos(kωt); the integrand is an even function of t so kω π/ω π/ω π/ω π/ω 2 1 t sin(kωt) dt = 2 t sin(kωt) dt = − t cos(kωt) +2 cos(kωt) dt −π/ω 0 kω 0 0 kω k+1 π/ω k+1 2π(−1) 2 2π(−1) = + sin(kωt) = kω 2 k2 ω2 0 kω 2 1 3 55. (a) sin4 x dx = − sin3 x cos x + sin2 x dx 4 4 1 3 1 1 = − sin3 x cos x + − sin x cos x + x + C 4 4 2 2 1 3 3 = − sin3 x cos x − sin x cos x + x + C 4 8 8
  • 12. January 27, 2005 11:45 L24-CH08 Sheet number 12 Page number 348 black 348 Chapter 8 π/2 π/2 π/2 1 4 (b) sin5 x dx = − sin4 x cos x + sin3 x dx 0 5 0 5 0 π/2 π/2 4 1 2 = − sin2 x cos x + sin x dx 5 3 0 3 0 π/2 8 8 =− cos x = 15 0 15 1 4 1 4 1 2 56. (a) cos5 x dx = cos4 x sin x + cos3 x dx = cos4 x sin x + cos2 x sin x + sin x + C 5 5 5 5 3 3 1 4 8 = cos4 x sin x + cos2 x sin x + sin x + C 5 15 15 1 5 (b) cos6 x dx = cos5 x sin x + cos4 x dx 6 6 1 5 1 3 = cos5 x sin x + cos3 x sin x + cos2 x dx 6 6 4 4 1 5 5 1 1 = cos5 x sin x + cos3 x sin x + cos x sin x + x + C, 6 24 8 2 2 π/2 1 5 5 5 cos5 x sin x + cos3 x sin x + cos x sin x + x = 5π/32 6 24 16 16 0 57. u = sinn−1 x, dv = sin x dx, du = (n − 1) sinn−2 x cos x dx, v = − cos x; sinn x dx = − sinn−1 x cos x + (n − 1) sinn−2 x cos2 x dx = − sinn−1 x cos x + (n − 1) sinn−2 x (1 − sin2 x)dx = − sinn−1 x cos x + (n − 1) sinn−2 x dx − (n − 1) sinn x dx, n sinn x dx = − sinn−1 x cos x + (n − 1) sinn−2 x dx, 1 n−1 sinn x dx = − sinn−1 x cos x + sinn−2 x dx n n 58. (a) u = secn−2 x, dv = sec2 x dx, du = (n − 2) secn−2 x tan x dx, v = tan x; secn x dx = secn−2 x tan x − (n − 2) secn−2 x tan2 x dx = secn−2 x tan x − (n − 2) secn−2 x (sec2 x − 1)dx = secn−2 x tan x − (n − 2) secn x dx + (n − 2) secn−2 x dx, (n − 1) secn x dx = secn−2 x tan x + (n − 2) secn−2 x dx, 1 n−2 secn x dx = secn−2 x tan x + secn−2 x dx n−1 n−1
  • 13. January 27, 2005 11:45 L24-CH08 Sheet number 13 Page number 349 black Exercise Set 8.2 349 (b) tann x dx = tann−2 x (sec2 x − 1) dx = tann−1 x sec2 x dx − tann−2 x dx 1 = tann−1 x − tann−2 x dx n−1 (c) u = xn , dv = ex dx, du = nxn−1 dx, v = ex ; xn ex dx = xn ex − n xn−1 ex dx 1 1 1 59. (a) tan4 x dx = tan3 x − tan2 x dx = tan3 x − tan x + dx = tan3 x − tan x + x + C 3 3 3 1 2 1 2 (b) sec4 x dx = sec2 x tan x + sec2 x dx = sec2 x tan x + tan x + C 3 3 3 3 (c) x3 ex dx = x3 ex − 3 x2 ex dx = x3 ex − 3 x2 ex − 2 xex dx = x3 ex − 3x2 ex + 6 xex − ex dx = x3 ex − 3x2 ex + 6xex − 6ex + C 60. (a) u = 3x, 1 1 1 2 u 2 x2 e3x dx = u2 eu du = u2 eu − 2 ueu du = u e − ueu − eu du 27 27 27 27 1 2 u 2 2 1 2 2 = u e − ueu + eu + C = x2 e3x − xe3x + e3x + C 27 27 27 3 9 27 √ (b) u = − x, 1 √ −1 xe− x dx = 2 u3 eu du, 0 0 u3 eu du = u3 eu − 3 u2 eu du = u3 eu − 3 u2 eu − 2 ueu du = u3 eu − 3u2 eu + 6 ueu − eu du = u3 eu − 3u2 eu + 6ueu − 6eu + C, −1 −1 2 u3 eu du = 2(u3 − 3u2 + 6u − 6)eu = 12 − 32e−1 0 0 61. u = x, dv = f (x)dx, du = dx, v = f (x); 1 1 1 x f (x)dx = xf (x) − f (x)dx −1 −1 −1 1 = f (1) + f (−1) − f (x) = f (1) + f (−1) − f (1) + f (−1) −1 62. (a) u dv = uv − v du = x(sin x + C1 ) + cos x − C1 x + C2 = x sin x + cos x + C2 ; the constant C1 cancels out and hence plays no role in the answer. (b) u(v + C1 ) − (v + C1 )du = uv + C1 u − v du − C1 u = uv − v du
  • 14. January 27, 2005 11:45 L24-CH08 Sheet number 14 Page number 350 black 350 Chapter 8 dx 63. u = ln(x + 1), dv = dx, du = , v = x + 1; x+1 ln(x + 1) dx = u dv = uv − v du = (x + 1) ln(x + 1) − dx = (x + 1) ln(x + 1) − x + C 3dx 2 64. u = ln(3x − 2), dv = dx, du = ,v = x − ; 3x − 2 3 2 2 1 ln(3x − 2) dx = u dv = uv − v du = x− ln(3x − 2) − x− dx 3 3 x − 2/3 2 2 = x− ln(3x − 2) − x − +C 3 3 1 1 65. u = tan−1 x, dv = x dx, du = 2 dx, v = (x2 + 1) 1+x 2 1 2 1 x tan−1 x dx = u dv = uv − v du = (x + 1) tan−1 x − dx 2 2 1 2 1 = (x + 1) tan−1 x − x + C 2 2 1 66. u = , dv = x dx, du = − x(ln x)2 dx, 1 1 v = ln x ln x 1 1 dx = 1 + dx. x ln x x ln x This seems to imply that 1 = 0, but recall that both sides represent a function plus an arbitrary constant; these two arbitrary constants will take care of the 1. 67. (a) u = f (x), dv = dx, du = f (x), v = x; b b b b f (x) dx = xf (x) − xf (x) dx = bf (b) − af (a) − xf (x) dx a a a a (b) Substitute y = f (x), dy = f (x) dx, x = a when y = f (a), x = b when y = f (b), b f (b) f (b) xf (x) dx = x dy = f −1 (y) dy a f (a) f (a) y (c) From a = f −1 (α) and b = f −1 (β) we get b bf (b) − af (a) = βf −1 (β) − αf −1 (α); then A1 β β f (b) f −1 (x) dx = f −1 (y) dy = f −1 (y) dy, a A2 α α f (a) x which, by Part (b), yields a= f –1(a) b=f –1(b) β b f −1 (x) dx = bf (b) − af (a) − f (x) dx α a f −1 (β) −1 −1 = βf (β) − αf (α) − f (x) dx f −1 (α) β f −1 (β) Note from the figure that A1 = f −1 (x) dx, A2 = f (x) dx, and α f −1 (α) A1 + A2 = βf −1 (β) − αf −1 (α), a “picture proof”.
  • 15. January 27, 2005 11:45 L24-CH08 Sheet number 15 Page number 351 black Exercise Set 8.3 351 68. (a) Use Exercise 67(c); 1/2 sin−1 (1/2) π/6 1 1 1 1 sin−1 x dx = sin−1 −0·sin−1 0− sin x dx = sin−1 − sin x dx 0 2 2 sin−1 (0) 2 2 0 (b) Use Exercise 67(b); e2 ln e2 2 2 ln x dx = e2 ln e2 − e ln e − f −1 (y) dy = 2e2 − e − ey dy = 2e2 − e − ex dx e ln e 1 1 EXERCISE SET 8.3 1 1. u = cos x, − u3 du = − cos4 x + C 4 1 1 2. u = sin 3x, u5 du = sin6 3x + C 3 18 1 1 1 3. sin2 5θ = (1 − cos 10θ) dθ = θ− sin 10θ + C 2 2 20 1 1 1 4. cos2 3x dx = (1 + cos 6x)dx = x+ sin 6x + C 2 2 12 1 1 5. sin3 aθ dθ = sin aθ(1 − cos2 aθ) dθ = − cos aθ − cos3 aθ + C (a = 0) a 3a 6. cos3 at dt = (1 − sin2 at) cos at dt 1 1 = cos at dt − sin2 at cos at dt = sin at − sin3 at + C (a = 0) a 3a 1 1 7. u = sin ax, u du = sin2 ax + C, a = 0 a 2a 8. sin3 x cos3 x dx = sin3 x(1 − sin2 x) cos x dx 1 1 = (sin3 x − sin5 x) cos x dx = sin4 x − sin6 x + C 4 6 9. sin2 t cos3 t dt = sin2 t(1 − sin2 t) cos t dt = (sin2 t − sin4 t) cos t dt 1 1 = sin3 t − sin5 t + C 3 5 10. sin3 x cos2 x dx = (1 − cos2 x) cos2 x sin x dx 1 1 = (cos2 x − cos4 x) sin x dx = − cos3 x + cos5 x + C 3 5 1 1 1 1 11. sin2 x cos2 x dx = sin2 2x dx = (1 − cos 4x)dx = x− sin 4x + C 4 8 8 32
  • 16. January 27, 2005 11:45 L24-CH08 Sheet number 16 Page number 352 black 352 Chapter 8 1 1 12. sin2 x cos4 x dx = (1 − cos 2x)(1 + cos 2x)2 dx = (1 − cos2 2x)(1 + cos 2x)dx 8 8 1 1 1 1 = sin2 2x dx + sin2 2x cos 2x dx = (1 − cos 4x)dx + sin3 2x 8 8 16 48 1 1 1 = x− sin 4x + sin3 2x + C 16 64 48 1 1 1 13. sin 2x cos 3x dx = (sin 5x − sin x)dx = − cos 5x + cos x + C 2 10 2 1 1 1 14. sin 3θ cos 2θdθ = (sin 5θ + sin θ)dθ = − cos 5θ − cos θ + C 2 10 2 1 1 15. sin x cos(x/2)dx = [sin(3x/2) + sin(x/2)]dx = − cos(3x/2) − cos(x/2) + C 2 3 3 16. u = cos x, − u1/3 du = − cos4/3 x + C 4 π/2 π/2 17. cos3 x dx = (1 − sin2 x) cos x dx 0 0 π/2 1 2 = sin x − sin3 x = 3 0 3 π/2 π/2 π/2 1 1 18. sin2 (x/2) cos2 (x/2)dx = sin2 x dx = (1 − cos 2x)dx 0 4 0 8 0 π/2 1 1 = x− sin 2x = π/16 8 2 0 π/3 π/3 π/3 1 1 19. sin4 3x cos3 3x dx = sin4 3x(1 − sin2 3x) cos 3x dx = sin5 3x − sin7 3x =0 0 0 15 21 0 π π π 1 1 1 20. cos2 5θ dθ = (1 + cos 10θ)dθ = θ+ sin 10θ =π −π 2 −π 2 10 −π π/6 π/6 π/6 1 1 1 21. sin 4x cos 2x dx = (sin 2x + sin 6x)dx = − cos 2x − cos 6x 0 2 0 4 12 0 = [(−1/4)(1/2) − (1/12)(−1)] − [−1/4 − 1/12] = 7/24 2π 2π 2π 1 1 1 1 22. sin2 kx dx = (1 − cos 2kx)dx = x− sin 2kx =π− sin 4πk (k = 0) 0 2 0 2 2k 0 4k 1 1 23. tan(2x − 1) + C 24. − ln | cos 5x| + C 2 5 25. u = e−x , du = −e−x dx; − tan u du = ln | cos u| + C = ln | cos(e−x )| + C 1 1 26. ln | sin 3x| + C 27. ln | sec 4x + tan 4x| + C 3 4
  • 17. January 27, 2005 11:45 L24-CH08 Sheet number 17 Page number 353 black Exercise Set 8.3 353 √ 1 √ √ 28. u = x, du = √ dx; 2 sec u du = 2 ln | sec u + tan u| + C = 2 ln sec x + tan x + C 2 x 1 29. u = tan x, u2 du = tan3 x + C 3 1 1 30. tan5 x(1 + tan2 x) sec2 x dx = (tan5 x + tan7 x) sec2 x dx = tan6 x + tan8 x + C 6 8 1 1 31. tan 4x(1 + tan2 4x) sec2 4x dx = (tan 4x + tan3 4x) sec2 4x dx = tan2 4x + tan4 4x + C 8 16 1 1 32. tan4 θ(1 + tan2 θ) sec2 θ dθ = tan5 θ + tan7 θ + C 5 7 1 1 33. sec4 x(sec2 x − 1) sec x tan x dx = (sec6 x − sec4 x) sec x tan x dx = sec7 x − sec5 x + C 7 5 1 2 34. (sec2 θ − 1)2 sec θ tan θdθ = (sec4 θ − 2 sec2 θ + 1) sec θ tan θdθ = sec5 θ − sec3 θ + sec θ + C 5 3 35. (sec2 x − 1)2 sec x dx = (sec5 x − 2 sec3 x + sec x)dx = sec5 x dx − 2 sec3 x dx + sec x dx 1 3 = sec3 x tan x + sec3 x dx − 2 sec3 x dx + ln | sec x + tan x| 4 4 1 5 1 1 = sec3 x tan x − sec x tan x + ln | sec x + tan x| + ln | sec x + tan x| + C 4 4 2 2 1 5 3 = sec3 x tan x − sec x tan x + ln | sec x + tan x| + C 4 8 8 36. [sec2 x − 1] sec3 x dx = [sec5 x − sec3 x]dx 1 3 = sec3 x tan x + sec3 x dx − sec3 x dx (equation (20)) 4 4 1 1 = sec3 x tan x − sec3 x dx 4 4 1 1 1 = sec3 x tan x − sec x tan x − ln | sec x + tan x| + C (equation (20), (22)) 4 8 8 1 1 37. sec2 t(sec t tan t)dt = sec3 t + C 38. sec4 x(sec x tan x)dx = sec5 x + C 3 5 1 39. sec4 x dx = (1 + tan2 x) sec2 x dx = (sec2 x + tan2 x sec2 x)dx = tan x + tan3 x + C 3 40. Using equation (20), 1 3 sec5 x dx = sec3 x tan x + sec3 x dx 4 4 1 3 3 = sec3 x tan x + sec x tan x + ln | sec x + tan x| + C 4 8 8
  • 18. January 27, 2005 11:45 L24-CH08 Sheet number 18 Page number 354 black 354 Chapter 8 41. u = 4x, use equation (19) to get 1 1 1 1 1 tan3 u du = tan2 u + ln | cos u| + C = tan2 4x + ln | cos 4x| + C 4 4 2 8 4 1 42. Use equation (19) to get tan4 x dx = tan3 x − tan x + x + C 3 √ 2 2 43. tan x(1 + tan2 x) sec2 x dx = tan3/2 x + tan7/2 x + C 3 7 2 44. sec1/2 x(sec x tan x)dx = sec3/2 x + C 3 π/8 π/8 1 45. (sec2 2x − 1)dx = tan 2x − x = 1/2 − π/8 0 2 0 π/6 π/6 1 46. sec2 2θ(sec 2θ tan 2θ)dθ = sec3 2θ = (1/6)(2)3 − (1/6)(1) = 7/6 0 6 0 47. u = x/2, π/4 1 π/4 √ 2 tan5 u du = tan4 u − tan2 u − 2 ln | cos u| = 1/2 − 1 − 2 ln(1/ 2) = −1/2 + ln 2 0 2 0 1 π/4 1 π/4 √ 48. u = πx, sec u tan u du = sec u = ( 2 − 1)/π π 0 π 0 1 1 49. (csc2 x − 1) csc2 x(csc x cot x)dx = (csc4 x − csc2 x)(csc x cot x)dx = − csc5 x + csc3 x + C 5 3 cos2 3t 1 1 50. · dt = csc 3t cot 3t dt = − csc 3t + C sin2 3t cos 3t 3 cos x 1 51. (csc2 x − 1) cot x dx = csc x(csc x cot x)dx − dx = − csc2 x − ln | sin x| + C sin x 2 1 52. (cot2 x + 1) csc2 x dx = − cot3 x − cot x + C 3 2π 2π 1 53. (a) sin mx cos nx dx = [sin(m + n)x + sin(m − n)x]dx 0 2 0 2π cos(m + n)x cos(m − n)x = − − 2(m + n) 2(m − n) 0 2π 2π but cos(m + n)x = 0, cos(m − n)x = 0. 0 0 2π 1 2π (b) cos mx cos nx dx = [cos(m + n)x + cos(m − n)x]dx; 0 2 0 since m = n, evaluate sin at integer multiples of 2π to get 0. 2π 1 2π (c) sin mx sin nx dx = [cos(m − n)x − cos(m + n)x] dx; 0 2 0 since m = n, evaluate sin at integer multiples of 2π to get 0.